Você está na página 1de 61

Prof.

Rodrigo Lins

Mecânica F = 1g . cm/s2 = 1dyn (dyna); onde: 1dyn = 105N

• Outra Unidade:
F = 1 kilograma-força (1kgf) ; onde: 1kgf = 9,8N
• Os princípios fundamentais da Dinâmica
4. O Princípio da Ação e da Reação (3ª Lei de
1. Introdução
Newton).
A Dinâmica é a parte da Mecânica que estuda os
A toda força de ação corresponde uma de reação,
movimentos e as causas que os produzem ou os
de modo que essas forças têm sempre mesma
modificam.
intensidade, mesma direção e sentidos opostos,
estando aplicadas em corpos diferentes.
2. Princípio da inércia (primeira lei de Newton)
Um ponto material é chamado isolado quando
não existem forças atuando nele ou quando as forças
aplicadas ao ponto têm soma vetorial nula. O princípio
da inércia (ou primeira lei de Newton) estabelece:

AFA – EFOMM – EEAR| FÍSICA


Um ponto material isolado mecanicamente está
em repouso ou em movimento retilíneo uniforme.
Isso significa que um ponto material isolado
possui velocidade vetorial constante.

Ao caminhar, uma pessoa age no chão,


empurrando-o "para trás". Este, por sua vez, reage na
pessoa, empurrando-a "para a frente". Observemos,
nesse caso, que a ação está aplicada no solo,
enquanto a reação está aplicada na pessoa.

5. Classes de forças
Quanto ao modo como são exercidas, as forças
podem ser divididas em duas classes: forças de
Ex: Por inércia, o cavaleiro tende a prosseguir com a contato e forças de campo.
sua velocidade.
5.1 Forças de contato
São forças que existem quando duas superfícies
entram em contato. Quando empurramos um bloco
contra uma parede, há forças de contato entre o bloco
e a parede. Analogamente aparecem forças de contato
entre uma mesa e um corpo apoiado sobre ela.

3. Princípio fundamental da Dinâmica (Segunda


Lei de Newton).
A resultante das forças aplicadas a um ponto
material é igual ao produto de sua massa pela
aceleração adquirida:

  Exemplo:
F = ma
Força de reação normal (N)

5.2 Forças de campo


• Unidades no (SI):
Massa: quilograma (kg) São forças que os corpos exercem mutuamente,
Aceleração: m/s2 ainda que estejam distantes uns dos outros. A Terra
F = 1 kg . m/s2 = 1N (Newton) atrai corpos, exercendo neles forças de campo.
No espaço, em torno da Terra, existe o campo de
• Unidades no (CGS): forças chamado campo gravitacional terrestre. A força
Massa: grama (g) com que a Terra atrai um corpo (peso do corpo) se
Aceleração: cm/s2 deve à interação entre o campo gravitacional terrestre
e a massa do corpo.
1 OS.:0424/13-Juliana
Prof. Rodrigo Lins

Reciprocamente, o corpo atrai a Terra devido à Em regime elástico, a deformação sofrida por
interação entre o campo gravitacional do corpo e a uma mola é diretamente proporcional à intensidade
massa da Terra. da força que a provoca. A expressão matemática da
Assim, o campo desempenha o papel de Lei de Hooke é dada a seguir:
transmissor de interações entre corpos. Analogamente,
em torno de cada corpo eletrizado existe um campo
de forças denominado campo elétrico. F = K∆x

em que:
F → é a intensidade da força deformadora;
K → é a constante de proporcionalidade;
Δx → é a deformação (alongamento ou encurtamento
sofrido pela mola).

• Força peso (P) • Gráfico (F x Δx)


AFA – EFOMM – EEAR| FÍSICA

Quando um corpo está em movimento sob ação


exclusiva de seu peso P, ele adquire uma aceleração
denominada aceleração da gravidade g. Sendo m a
massa do corpo.
   
FR = ma ⇒ P = mg
↓ ↓
  F
P g tg=
θ = K
∆x
Em módulo, temos:

Força de Atrito
P = mg
1. Introdução
Unidade (SI): O atrito é um fenômeno de grande importância
Massa: quilograma (kg) no acontecimento de determinados fatos em nossa
Aceleração da gravidade próximo a superfície: g ≈ 9,8 m/s2 vida diária. Se, por um lado, apresenta um caráter útil,
P = 1 kg . m/s2 = 1N (Newton) por outro, revela um caráter indesejável. Comecemos
analisando a força de atrito de escorregamento entre
6. Deformações em sistemas elásticos (Lei de sólidos. O atrito é denominado dinâmico quando
Hooke) há movimento relativo entre os corpos em contato.
Quando não há movimento, o atrito é denominado
Consideremos a figura abaixo, em que uma mola estático.
de massa desprezível tem uma de suas extremidades
fixas. O comprimento da mola na situação A é seu
comprimento natural (x0). Portanto, a mola não está
deformada. Na situação B, uma força F foi aplicada
à extremidade livre da mola, provocando nela uma
deformação (alongamento) Δx.
Na situação C, F foi suprimida e a mola recobrou
seu comprimento natural (x0).

Uma superfície qualquer, por mais bem polida que


seja, sempre apresenta irregularidades: saliências"
reentrâncias, altos e baixos, enfim, asperezas.

As forças FBS e FSB que B e S trocam na região


de contato são forças de atrito e constituem um par
ação-reação (3ª Lei de Newton).

OS.:0424/13-Juliana
2
Prof. Rodrigo Lins

• Graficamente, a intensidade da força de atrito


recebida por um corpo em função da intensidade
da força que o solicita ao escorregamento é dada
conforme o diagrama seguinte:

2. Força de atrito estático (FATE)


Considere um corpo em repouso sobre uma
superfície horizontal. Vamos aplicar no corpo uma
força F que tende a deslocar na direção horizontal.
Enquanto o corpo estiver em repouso à medida
que a intensidade da força solicitadora F aumenta
intensidade da força de atrito também aumenta, de
modo que F e FAT se equilibram. • Resultantes tangencial e centrípeta.

AFA – EFOMM – EEAR| FÍSICA


1. Componentes da força resultante
Devemos recordar que a força resultante é o
resultado de uma adição vetorial, ou seja, é a soma
vetorial de todas as forças que atuam na partícula.
Assim, a força de atrito FAT tem intensidade
A resultante P pode ser decomposta em duas
igual à da força solicitadora F enquanto não houver
movimento. Se F continuar crescendo, FAT também direções perpendiculares entre si: uma tangencial
crescerá até atingir um valor máximo e o corpo ficará e outra normal à trajetória. Essa decomposição é
na iminência de movimento. usualmente feita quando conveniente. Decompondo
A máxima intensidade da força de atrito estático, P, obtemos a configuração a seguir:
e que corresponde à iminência de movimento, é
dada por:

fat(máx) = µeFN

μE → Coeficiente de atrito estático.


FN → Força de reação Normal.

3. Força de atrito dinâmico ou cinético (FATD)


Enquanto o bloco estava em repouso, o atrito
era chamado de estático. Agora porém, receberá a
2
denominação de atrito cinético (ou dinâmico). F= Ft2 + Fcp2

Para Ft e FCP atribuímos as denominações


componente tangencial e componente centrípeta
respectivamente.
Sendo F > FATC , o bloco entra em movimento e, • O termo "centrípeta” advém do fato de a
nessa situação, o atrito recebido do plano de apoio é componente FCP estar, a cada instante, dirigida
cinético. Matematicamente, temos: para o centro de curvatura da trajetória.

Fatc = µcFn
2. A componente tangencial (Ft)

μC → Coeficiente de atrito cinético. Assim, a intensidade da componente tangencial


FN → Força de reação Normal. da força resultante pode ser expressa por:

NOTAS: →
|Ft| = m |α|
• μ → é uma grandeza adimensional.
• μE > μC.
• A força de atrito dinâmico independe da velocidade A direção de Ft é sempre a da tangente à trajetória
com que o corpo desliza sobre a superfície e em cada instante. Por isso, é a mesma da velocidade
também independe da área de contato entre o vetorial, que também é tangente à trajetória em cada
corpo e a superfície. instante.
3 OS.:0424/13-Juliana
Prof. Rodrigo Lins

NOTA: Casos Particulares

Trem-bala: rapidez e conforto


Os trens-bala utilizados na Europa e no Japão
trafegam ao longo das ferrovias com velocidades
da ordem de 500km/h. Na fase de arrancada, a)
que se sucede à partida de uma estação, a força
resultante sobre eles deve admitir uma componente
tangencial no sentido do movimento, o que provoca a
intensificação da velocidade vetorial.
AFA – EFOMM – EEAR| FÍSICA

→ →
Fg = Fcp

3. A componente centrípeta (Fcp)


Sem força centrípeta, corpo nenhum pode b)
manter-se em trajetória curvilínea.A componente
centrípeta da força resultante (Fcp) tem por função
variar a direção da velocidade vetorial (V) da partícula
móvel.

→ →
Fat = Fcp

Aplicando a 2a Lei de Newton, podemos escrever


4. As componentes tangencial e centrípeta nos
que:
principais movimentos
  v2 • M.R.U.
Fcp ma
= cp onde : acp
R A resultante total é nula.

• M.R.U.V.
Assim, a intensidade da componente centrípeta A resultante total é tangencial.
da força resultante fica determinada por:
• M.C.U.
 mv 2  A resultante é centrípeta.
Fcp = ou Fcp = mω2R
R
• M.C.U.V.
A resultante total admite duas componentes:
a tangencial e a centrípeta.
OS.:0424/13-Juliana
4
Prof. Rodrigo Lins

5. Força centrífuga podemos afirmar que, no movimento subsequente,


Consideremos um conjunto moto-piloto a velocidade do foguete mantém:
descrevendo uma curva circular em movimento a) módulo nulo.
uniforme. Nesse caso, em relação a um referencial b) módulo constante e direção constante.
ligado ao solo (referencial inercial), a resultante das
c) módulo constante e direção variável.
forças no corpo do piloto é radial e dirigida para o
d) módulo variável e direção constante.
centro da curva, sendo denominada centrípeta.
Em relação a um referencial ligado à moto e) módulo variável e direção variável.
(referencial acelerado), entretanto, o piloto está
em repouso e, por isso, a resultante das forças que 02. Um pêndulo simples no interior de um avião tem
agem em seu corpo deve ser nula. Isso significa que, a extremidade superior do fio fixa no teto. Quando
em relação a esse referencial, deve ser considerada o avião está parado o pêndulo fica na posição
uma força que equilibra a resultante centrípeta. A
vertical.
equilibrante da força centrípeta é, portanto, uma força
também radial, porém, dirigida para fora da trajetória, Durante a corrida para a decolagem a aceleração

AFA – EFOMM – EEAR| FÍSICA


sendo denominada centrífuga (Fcf) do avião foi constante e o pêndulo fez um ângulo θ
com a vertical. Sendo g a aceleração da gravidade,
a relação entre a, θ e g é:
a) g2 = (1 – sec2θ)a2
b) g2 = (a2 + g2)sen2θ
c) a = g tg θ
d) a = g sen θ cos θ
e) g2 = a2sen2θ + g2cos2θ

03. Um trenó de massa igual a 10,0kg é puxado por


   mv 2 uma criança por meio de uma corda, que forma um
Fcf =Fcp ⇒ Fcp =
R ângulo de 45° com a linha do chão. Se a criança
aplicar uma força de 60,0N ao longo da corda,
A força centrífuga é uma força de inércia que
é introduzida para justificar o equilíbrio de um corpo considerando g = 9,81m/s2, indique a alternativa
em relação a um referencial acelerado quando esse que contém afirmações corretas:
corpo descreve trajetórias curvilíneas em relação a a) As componentes horizontal e vertical da força
um referencial inercial.
aplicada pela criança são iguais e valem
30,0N.
NOTA: b) As componentes são iguais e valem 42,4N.
Um piloto de Fórmula 1 tem a musculatura do c) A força vertical é tão grande que ergue o trenó.
pescoço bastante solicitada ao fazer uma curva. Isso d) A componente horizontal da força vale 42,4N
se deve à força centrífuga, que "puxa" sua cabeça e a vertical vale 30,0N.
para fora da trajetória. Alguns amenizam esse efeito
adaptando elásticos, que conectam o capacete aos e) A componente vertical é 42,4N e a horizontal
ombros. Deve-se entender, entretanto, que a força vale 30,0N.
centrífuga não existe para quem vê a corrida parado
em relação ao solo; ela é definida em relação ao 04. Analise as afirmações a seguir e assinale a
carro, que é um referencial acelerado. alternativa correta:
I. Massa e peso são grandezas proporcionais.
II. Massa e peso variam inversamente.
Exercícios de Fixação III. A massa é uma grandeza escalar e o peso
uma grandeza vetorial.

01. O motor de um foguete de massa m é acionado em a) somente a I é correta.


um instante em que ele se encontra em repouso
b) I e II são corretas.
sob a ação da gravidade (g constante). O motor
exerce uma força constante perpendicular à c) I e III são corretas.
força exercida pela gravidade. Desprezando-se a d) todas são incorretas.
resistência do ar e a variação da massa do foguete, e) todas são corretas.
5 OS.:0424/13-Juliana
Prof. Rodrigo Lins

05. Um homem tenta levantar uma caixa de 5kg, que 08. A respeito de uma partícula em equilíbrio, examine
está sobre uma mesa, aplicando uma força vertical as proposições abaixo:
de 10N. Nesta situação, o valor da força que a I. Não recebe a ação de forças.
mesa aplica na caixa é: Adote: g = 10m/s2 II. Descreve trajetória retilínea.
III. Pode estar em repouso.
IV. Pode ter altas velocidades.
São corretas:
a) todas;
b) apenas I e II;
c) apenas I e III;
d) apenas III e IV;
e) apenas I, III e IV.
a) 0N
b) 5N
AFA – EFOMM – EEAR| FÍSICA

09. Considere a figura abaixo.


c) 10N
d) 40N
e) 50N

06. Dois corpos equilibram-se quando colocados cada


um num dos pratos de uma balança de braços
iguais. Em seguida, um deles é acelerado por
uma força resultante de 2N. Verifica-se então que
sua velocidade varia de 8m/s cada 2 segundos. A
massa do corpo que ficou na balança é:
a) 1/4kg Dadas as forças F1, F2‚ F3, o módulo de sua
b) 1/2kg resultante, em N, é:
c) 1kg a) 30
d) 2kg b) 40
e) 4kg c) 50
d) 70
e) 80
07. (Cesgranrio-RJ) Uma bolinha descreve uma
trajetória circular sobre uma mesa horizontal sem
atrito, presa a um prego por um cordão (figura 10. Uma gota d'água cai no ar. A força de resistência do
seguinte). ar sobre a gota d'água é proporcional à velocidade
da gota de acordo com o gráfico abaixo.
Dado: g = 10m/s2.

Quando a bolinha passa pelo ponto P, o cordão


que a prende ao prego arrebenta. A trajetória que
a bolinha então descreve sobre a mesa é:

Uma gota de água de 0,10g passará a ter


velocidade de queda constante quando tiver
atingido a velocidade, em m/s, de:
a) 1 d) 7
b) 3 e) 9
c) 5

OS.:0424/13-Juliana
6
Prof. Rodrigo Lins

11. A respeito das leis de Newton, são feitas três 13. (Cesgranrio-RJ) Um pedaço de giz é lançado
afirmativas: horizontalmente de uma altura H. Desprezando-se

I. A força resultante necessária para acelerar, a influência do ar, a figura que melhor representa
uniformemente, um corpo de massa 4,0kg, de a(s) força(s) que age(m) sobre o giz é:
10m/s para 20m/s, em uma trajetória retilínea,
em 5,0s, tem módulo igual a 8,0N.
II. Quando uma pessoa empurra uma mesa, ela
não se move, podemos concluir que a força
de ação é anulada pela força de reação.
III. Durante uma viagem espacial, podem-se
desligar os foguetes da nave que ela continua
a se mover.

AFA – EFOMM – EEAR| FÍSICA


Esse fato pode ser explicado pela primeira lei de
Newton.
Assinale: 14. Um corpúsculo desloca-se em movimento retilíneo
a) se todas as afirmativas estiverem corretas. e acelerado de modo que, num instante t, sua
b) se todas as afirmativas estiverem incorretas. velocidade é v. Sendo F e a, respectivamente,
c) se apenas as afirmativas I e II estiverem a força resultante e a aceleração no instante
corretas. referido, aponte a alternativa que traz um possível
d) se apenas as afirmativas I e III estiverem esquema para os vetores v, F e a.
corretas.
e) se apenas as afirmativas II e III estiverem
corretas.

12. Assinale a alternativa que apresenta o enunciado


da Lei da Inércia, também conhecida como
Primeira Lei de Newton.

a) Qualquer planeta gira em torno do Sol


descrevendo uma órbita elíptica, da qual o Sol
ocupa um dos focos.
b) Dois corpos quaisquer se atraem com uma
força proporcional ao produto de suas massas
e inversamente proporcional ao quadrado da
15. Complete corretamente a frase a seguir, relativa à
distância entre eles.
primeira lei de Newton: "Quando a força resultante,
c) Quando um corpo exerce uma força sobre
que atua numa partícula, for nula, então a partícula:
outro, este reage sobre o primeiro com uma
força de mesma intensidade e direção, mas a) estará em repouso ou em movimento retilíneo
de sentido contrário. uniforme".
d) A aceleração que um corpo adquire é b) poderá estar em movimento circular e
diretamente proporcional à resultante das uniforme".
forças que nele atuam, e tem mesma direção c) terá uma aceleração igual à aceleração da
e sentido dessa resultante. gravidade local".
e) Todo corpo continua em seu estado de repouso d) estará com uma velocidade que se modifica
ou de movimento uniforme em uma linha reta, com o passar do tempo".
a menos que sobre ele estejam agindo forças e) poderá estar em movimento uniformemente
com resultante não nula. retardado".
7 OS.:0424/13-Juliana
Prof. Rodrigo Lins

16. Em 21 de junho de 2004, a nave espacial 19. Um dinamômetro possui suas duas extremidades
"SpaceShipOne" realizou um fato memorável: foi o presas a duas cordas. Duas pessoas puxam as
primeiro veículo espacial concebido pela iniciativa cordas na mesma direção e sentidos opostos, com
privada a entrar em órbita em torno da Terra, em uma força de mesma intensidade F = 100N. Quanto
altura pouco superior a 100km. Durante o intervalo marcará o dinamômetro?
de tempo em que a nave alcançou sua máxima a) 200N
altitude, e com os motores praticamente desligados,
b) 0
seu piloto abriu um pacote de confeitos de chocolates
c) 100N
para vê-los flutuar no interior da nave.
d) 50N
Assinale a alternativa que apresenta corretamente e) 400N
a explicação da flutuação dos confeitos.
a) A gravidade é praticamente zero na altitude
20. No campeonato mundial de arco e flecha dois
indicada.
concorrentes discutem sobre a Física que está
b) Não há campo gravitacional fora da atmosfera
contida na arte do arqueiro. Surge então a seguinte
AFA – EFOMM – EEAR| FÍSICA

da Terra.
dúvida: quando o arco está esticado, no momento
c) A força gravitacional da Terra é anulada pela
do lançamento da flecha, a força exercida sobre a
gravidade do Sol e da Lua.
corda pela mão do arqueiro é igual à:
d) As propriedades especiais do material de que
é feita a nave espacial blindam, em seu interior, I. força exercida pela sua outra mão sobre a
o campo gravitacional da Terra. madeira do arco.
e) Nave e objetos dentro dela estão em "queda II. tensão da corda.
livre", simulando uma situação de ausência de III. força exercida sobre a flecha pela corda no
gravidade. momento em que o arqueiro larga a corda.

17. A respeito do conceito de inércia, pode-se dizer que: Neste caso:


a) todas as afirmativas são verdadeiras
a) inércia é uma força que mantém os objetos
b) todas as afirmativas são falsas
em repouso ou em movimento com velocidade
constante. c) somente I e III são verdadeiras
b) inércia é uma força que leva todos os objetos d) somente I e II são verdadeiras
ao repouso. e) somente II é verdadeira
c) um objeto de grande massa tem mais inércia
que um de pequena massa. 21. É frequente observarmos, em espetáculos ao ar
d) objetos que se movem rapidamente têm mais livre, pessoas sentarem nos ombros de outras
inércia que os que se movem lentamente. para tentar ver melhor o palco. Suponha que Maria
esteja sentada nos ombros de João que, por sua
vez, está em pé sobre um banquinho colocado no
18. Considere um veículo, como o representado
chão.
abaixo, em movimento retilíneo sobre um plano
horizontal. Pelo fato de estar acelerado para a Com relação à terceira lei de Newton, a reação ao
direita, um pêndulo preso ao seu teto desloca-se peso de Maria está localizada no:
em relação à posição de equilíbrio, formando um a) chão
ângulo com a vertical. b) banquinho
c) centro da Terra
d) ombro de João

22. Uma mola helicoidal de comprimento natural


20cm pende verticalmente quando é presa pela
extremidade superior. Suspendendo-se um
corpo de massa 200g pela extremidade inferior,
seu comprimento passa a ser 25cm. A constante
São conhecidos o ângulo , o módulo da aceleração
elástica da mola é:
da gravidade (g) e a massa da esfera (m) atada ao
fio ideal. Dado g = 10m/s2
a) Qual o módulo da aceleração a do veículo? a) 4,0N/m d) 4,0.102N/m
b) O módulo de a depende de m? b) 8,0N/m e) 5,0.102N/m
c) 4,0.101N/m
OS.:0424/13-Juliana
8
Prof. Rodrigo Lins

23. Quanto à figura abaixo, podemos afirmar que: 26. Os três corpos, A, B e C, representados na figura
abaixo têm massas iguais, m = 3,0kg.

O plano horizontal, onde se apóiam A e B, não


oferece atrito, a roldana tem massa desprezível
a) não existe atrito e a aceleração local da gravidade pode ser
b) a aceleração do corpo B é o dobro da considerada g = 10m/s2. A tração no fio que une
aceleração do corpo A os blocos A e B tem módulo:
c) a força normal do corpo A é o dobro da força

AFA – EFOMM – EEAR| FÍSICA


normal em B a) 10N d) 25N
d) a força que o fio exerce no corpo A é o dobro b) 15N e) 30N
da força que o fio exerce no corpo B c) 20N
e) a aceleração do corpo B é a metade da
aceleração do corpo A 27. Suponha que um comerciante inescrupuloso
aumente o valor assinalado pela sua balança,
empurrando sorrateiramente o prato para baixo
24. Dois corpos, A e B, de massas MA e MB, estão com uma força F de módulo 5,0N, na direção e
apoiados em uma superfície horizontal sem atrito. sentido indicados na figura.
Sobre eles são aplicadas forças iguais. A variação
de suas velocidades é dada pelo gráfico. Para os
corpos, é correto afirmar que:

Com essa prática, ele consegue fazer com que


uma mercadoria de massa 1,5kg seja medida por
essa balança como se tivesse massa de:
a) 3,0kg. d) 1,8kg.
b) 2,4kg. e) 1,7kg.
a) MA/MB = 4 c) 2,1kg.
b) MA/MB = 3
c) MA/MB = 1/3 28. Para vencer o atrito e deslocar um grande
d) MA/MB = 1/2 contêiner C, na direção indicada, é necessária uma
e) MA/MB = 2 força F = 500N. Na tentativa de movê-lo, blocos de
massa m = 15kg são pendurados em um fio, que é
esticado entre o contêiner e o ponto P na parede,
25. A mola da figura tem constante elástica 20N/m e
como na figura. Para movimentar o contêiner, é
encontra-se deformada de 20cm sob a ação do
preciso pendurar no fio, no mínimo, Obs: sen 45°
corpo A cujo peso é 5N. Nessa situação, a balança,
= cos 45° = 0,7 e tg 45° = 1
graduada em newtons, marca:
a) 1N
b) 2N
a) 1 bloco
c) 3N
b) 2 blocos
d) 4N
c) 3 blocos
e) 5N
d) 4 blocos
e) 5 blocos

9 OS.:0424/13-Juliana
Prof. Rodrigo Lins

29. Uma pessoa de massa m está no interior de um 32. Os corpos A e B de massas m A e m B ,


elevador de massa M, que desce verticalmente, respectivamente, estão interligados por um fio que
diminuindo sua velocidade com uma aceleração de passa pela polia, conforme a figura. A polia pode
módulo a. Se a aceleração local da gravidade é g, a girar livremente em torno de seu eixo. A massa do
força feita pelo cabo que sustenta o elevador é: fio e da polia são considerados desprezíveis.
a) (M+m)(g-a)
b) (M+m)(g+a)
c) (M+m)(a-g)
d) (M-m)(g+a)

30.

Se o sistema está em repouso é correto afirmar:


AFA – EFOMM – EEAR| FÍSICA

I. Se mA = mB, necessariamente existe atrito


entre o corpo B e o plano inclinado.
II. Independente de existir ou não atrito entre o
plano e o corpo B, deve-se ter mA = mB.
Um caminhão-tanque, após sair do posto, segue, III. Se não existir atrito entre o corpo B e o plano
com velocidade constante, por uma rua plana inclinado, necessariamente mA > mB.
que, num dado trecho, é plana e inclinada. O IV. Se não existir atrito entre o corpo B e o plano
módulo da aceleração da gravidade, no local, é inclinado, necessariamente mA > mB.
g = 10m/s2, e a massa do caminhão, 22t, sem
considerar a do combustível. É correto afirmar que
Está correta ou estão corretas:
o coeficiente de atrito dinâmico entre o caminhão
a) Somente I.
e a rua é:
b) Somente II .
a) µ = cot α. c) I e III.
b) µ = csc α.
d) I e IV.
c) µ = sen α.
e) Somente III.
d) µ = tan α.
e) µ = cos α.
33. Selecione a alternativa que preenche corretamente
as lacunas do texto abaixo, na ordem em que elas
31. Um fio, que tem suas extremidades presas aos
aparecem. Na sua queda em direção ao solo, uma
corpos A e B, passa por uma roldana sem atrito e
gota de chuva sofre o efeito da resistência do ar.
de massa desprezível. O corpo A, de massa 1,0kg,
Essa força de atrito é contrária ao movimento e
está apoiado num plano inclinado de 37° com a
aumenta com a velocidade da gota. No trecho
horizontal, suposto sem atrito. Adote g = 10m/s2,
sen 37° = 0,60 e cos 37° = 0,80. inicial da queda, quando a velocidade da gota é
pequena e a resistência do ar também, a gota está
animada de um movimento ........ . Em um instante
posterior, a resultante das forças exercidas sobre
a gota torna-se nula. Esse equilíbrio de forças
ocorre quando a velocidade da gota atinge o valor
que torna a força de resistência do ar igual, em
módulo, ........ da gota. A partir desse instante, a
gota ........ .
Para o corpo B descer com aceleração de a) acelerado - ao peso - cai com velocidade
2,0m/s2, o seu peso deve ser, em newtons, constante
a) 2,0 b) uniforme - à aceleração - cai com velocidade
b) 6,0 decrescente
c) 8,0 c) acelerado - ao peso - para de cair
d) 10 d) uniforme - à aceleração - para de cair
e) 20 e) uniforme - ao peso - cai com velocidade
decrescente
OS.:0424/13-Juliana
10
Prof. Rodrigo Lins

34. (Unicamp-SP) Um carro de massa 800kg, andando 37. Durante uma apresentação da Esquadrilha da
a 108km/h, freia bruscamente e pára em 5,0s. Fumaça, um dos aviões descreve a trajetória
circular da figura, mantendo o módulo de sua
a) Qual o módulo da desaceleração do carro, velocidade linear sempre constante.
admitida constante? Sobre o descrito são feitas as seguintes afirmações:
b) Qual a intensidade da força de atrito que a I. A força com a qual o piloto comprime o
pista aplica sobre o carro durante a freada? assento do avião varia enquanto ele percorre
a trajetória descrita.
II. O trabalho realizado pela força centrípeta que
35. Um bloco de borracha de massa 5,0kg está em age sobre o avião é nulo em qualquer ponto
repouso sobre uma superfície plana e horizontal. da trajetória descrita.
O gráfico representa como varia a força de atrito III. Entre os pontos A e B da trajetória descrita
pelo avião não há impulso devido à ação da
sobre o bloco quando sobre ele atua uma força F força centrípeta.
de intensidade variável paralela à superfície.

AFA – EFOMM – EEAR| FÍSICA


O coeficiente de atrito estático entre a borracha e
a superfície, e a aceleração adquirida pelo bloco Somente está correto o que se lê em:
a) I
quando a intensidade da força F atinge 30N são,
b) II
respectivamente, iguais a:
c) III
a) 0,3; 4,0m/s2 d) II e III
b) 0,2; 6,0m/s2 e) I e II

c) 0,3; 6,0m/s2
d) 0,5; 4,0 m/s2 38. Num dado instante, se a velocidade do fluxo
sanguíneo num ponto do eixo central da aorta
e) 0,2; 3,0 m/s2
é de 28cm/s e o raio desse vaso é 1cm, então a
velocidade em um ponto que dista 0,5cm desse
eixo é, em centímetros por segundo, igual a:
36. Um garoto arremessa verticalmente para cima uma
pedra, que passa a mover-se sob a ação exclusiva a) 19 d) 25
b) 21 e) 27
do campo gravitacional terrestre. A influência do
c) 23
ar é desprezível. A alternativa que representa
corretamente os vetores força resultante na 39. No modelo clássico do átomo de hidrogênio, do
pedra (F), aceleração resultante (a) e velocidade físico dinamarquês Niels Bohr, um elétron gira em
instantânea (v), em dado instante do movimento torno de um próton com uma velocidade constante
de subida, é: de 2 × 106 m/s e em uma órbita circular de raio
igual a 5 × 10-11m. Se o elétron possui massa
  
a) ↑ F ↑ a ↑ v 9 × 10-31kg, a força centrípeta sobre ele é de:
  
b) ↑ F ↓ a ↑ v a) 7,2 × 10–14N.
  
c) ↓ F ↓ a ↑ v b) 3,6 × 10–14N.
   c) 8,0 × 10–10N.
d) ↑ F ↓ a ↓ v
   d) 7,2 × 10–8N.
e) ↓ F ↓ a ↓ v e) 3,6 × 10–8N.
11 OS.:0424/13-Juliana
Prof. Rodrigo Lins

40. Um fio, de comprimento L, prende um corpo, de 42. Considere o esquema abaixo, em que estão
peso P e dimensões desprezíveis, ao teto. representados um elevador E de massa igual a
1,0 . 103 kg (incluída a massa do seu conteúdo), um
contrapeso B de massa igual a 5,0 . 102 kg e um
motor elétrico M que exerce no cabo conectado em
E uma força vertical constante F. Os dois cabos têm
massas desprezíveis, são flexíveis e inextensíveis
e as polias são ideais. No local, a influência do ar é
desprezível e adota- se g = 10m/s2.

Deslocado lateralmente, o corpo recebe um


impulso horizontal e passa a descrever um
AFA – EFOMM – EEAR| FÍSICA

movimento circular uniforme num plano horizontal,


de acordo com a figura acima.
A força resultante centrípeta sobre o corpo tem
intensidade: Se o elevador está acelerado para cima, com
a) T aceleração de módulo 0,20m/s2, a intensidade de F é:
b) P a) 4,7 . 103N
c) T–P b) 5,0 . 103N
d) T cos θ c) 5,2 . 103N
e) T sen θ d) 5,3 . 103N
e) 5,5 . 103N

41. Sobre o bloco A, de massa 2,0kg, atua a força


vertical F. O bloco B, de massa 4,0kg, é ligado ao
A por um fio inextensível, de massa desprezível e Exercícios Propostos
alta resistência à tração. Adote g = 10m/s2.

01. Um caminhão-guincho em movimento retilíneo


numa pista horizontal tem aceleração constante de
intensidade a. Ele transporta uma carga de massa M
sustentada por uma corda leve presa em sua traseira.
Nessas condições, o pêndulo, constituído pela carga
e a corda, permanece deslocado em um ângulo em
relação à vertical, conforme representa a figura:

Considere as proposições:
I. Se F = 60N, o sistema está em equilíbrio e a
tração no fio é 50N.
II. Se F = 120N, o sistema está em movimento
Sendo g a intensidade da aceleração da gravidade,
acelerado e a tração no fio é 40N.
III. Se F = 0, o sistema tem uma aceleração de 1 3 , aponte a alternativa que traz o
sen
= θ e=θ
10m/s2 e a tração no fio é nula. 2 2
valor correto de a:
IV. Se F = 12N, o sistema está em movimento
acelerado e a tração no fio é 8,0N. 2 3
a) g d) g
a) Apenas IV está correta. 3 2
b) Todas estão corretas. 1
b) g e) 3g
c) Apenas I está correta. 2
d) Apenas I, II e III estão corretas. 3
c) g
e) Apenas III e IV estão corretas. 3

OS.:0424/13-Juliana
12
Prof. Rodrigo Lins

02. Na montagem do esquema, os blocos A e B têm 05. Leia a tirinha abaixo:


pesos iguais a 100N cada um:

A indicação do dinamômetro ideal, que está


graduado em newtons, é de:
a) 400N
b) 200N
c) 100N
d) 50N

AFA – EFOMM – EEAR| FÍSICA


e) zero.

03. O bloco da figura, de massa m = 4,0kg, desloca-se


sob a ação de uma força horizontal constante de
intensidade F. A mola ideal, ligada ao bloco, tem
comprimento natural (isto é, sem deformação)
L0 = 14,0cm e constante elástica K = 160N/m.
Papai-Noel, o personagem da tirinha, é
reconhecidamente bastante opulento e
rechonchudo.

Suponha que ele esteja na Terra, na Lapônia, e


Desprezando-se as forças de atrito e sabendo-se que a balança utilizada se encontre em repouso,
que as velocidades escalares do bloco em A e B apoiada sobre o solo horizontal. Considere que,
são, respectivamente, iguais a 4,0m/s e 6,0m/s, na situação de repouso, Papai-Noel exerça sobre
qual é, em centímetros, o comprimento da mola a plataforma da balança uma compressão de
durante o movimento?
intensidade 1 200N.

A respeito do descrito, são feitas as seguintes


04. A figura abaixo representa o corte de um dos afirmações:
compartimentos de um foguete, que acelera
I. O peso do Papai-Noel, na Terra, tem
verticalmente para cima nas proximidades da
intensidade 1200N.
Terra.
II. A plataforma da balança exerce sobre
Papai-Noel uma força de intensidade 1 200N.
III. Papai-Noel exerce no centro de massa da
Terra uma força atrativa de intensidade menor
que 1 200N.
IV. O peso de Papai-Noel e a força que a
plataforma da balança exerce sobre ele
constituem entre si um par ação-reação.

É (são) verdadeira(s):
a) somente I e II
No teto do compartimento, está fixado um
b) somente II e III
dinamômetro ideal, que tem preso a si um bloco de
massa 4,0kg. Adotando |g| = 10m/s2 e admitindo c) somente I, II e III
que a indicação do dinamômetro seja 60N, d) somente I, III e IV
determine o módulo da aceleração do foguete. e) todas as afirmativas
13 OS.:0424/13-Juliana
Prof. Rodrigo Lins

06. (UFPE-mod.) Uma mola de constante elástica Sendo o piso plano e as forças de atrito
K = 1,5 . 103N/m é montada horizontalmente desprezíveis, o módulo da força F e o da força de
em um caminhão, ligando um bloco B de massa tração na corrente são, em N, respectivamente:
m = 30kg a um suporte rígido S. A superfície de a) 70 e 20.
contato entre o bloco B e a base C é perfeitamente b) 70 e 40.
lisa. Observa-se que, quando o caminhão se c) 70 e 50.
desloca sobre uma superfície plana e horizontal d) 60 e 20.
com aceleração a, dirigida para a direita, a mola e) 60 e 50.
sofre uma compressão Δx = 10cm. Determine o
módulo de a em m/s2.
09. Na montagem representada na figura, o fio é
inextensível e de massa desprezível; a polia pode
girar sem atrito em torno de seu eixo, tendo inércia
de rotação desprezível; as massas dos blocos A
e B valem, respectivamente, mA e mB; inexiste
AFA – EFOMM – EEAR| FÍSICA

atrito entre o bloco A e o plano horizontal em que


se apoia e a influência do ar é insignificante:

07. Na figura abaixo, os blocos A e B têm massas


mA = 6,0kg e mB = 2,0kg e, estando apenas
encostados entre si, repousam sobre um plano
horizontal perfeitamente liso.

Em determinado instante, o sistema é abandonado


à ação da gravidade. Assumindo para o módulo
da aceleração da gravidade o valor g, determine:
a) o módulo da aceleração do sistema;
A partir de um dado instante, exerce-se em A uma b) a intensidade da força que traciona o fio.
força horizontal F, de intensidade igual a 16N.
Desprezando a influência do ar, calcule: 10. O dispositivo esquematizado na figura é uma
a) o módulo da aceleração do conjunto. Máquina de Atwood. No caso, não há atritos, o fio
b) a intensidade das forças que A e B trocam é inextensível e desprezam-se sua massa e a da
entre si na região de contato. polia.

08. (FGV-SP) Dois carrinhos de supermercado, A e B,


podem ser acoplados um ao outro por meio de uma
pequena corrente de massa desprezível, de modo
que uma única pessoa, em vez de empurrar dois
carrinhos separadamente, possa puxar o conjunto
pelo interior do supermercado. Um cliente aplica
uma força horizontal constante de intensidade F
sobre o carrinho da frente, dando ao conjunto uma Supondo que os blocos A e B tenham massas
aceleração de intensidade 0,5 m/s2. respectivamente iguais a 3,0kg e 2,0kg e que
|g| = 10 m/s2, determine:
a) o módulo da aceleração dos blocos;
b) a intensidade da força de tração estabelecida
no fio;
c) a intensidade da força de tração estabelecida
na haste de sustentação da polia.

OS.:0424/13-Juliana
14
Prof. Rodrigo Lins

11. Um homem de massa 60kg acha-se de pé 14. Um garoto de massa igual a 40,0kg parte do repouso
sobre uma balança graduada em newtons. Ele do ponto A do escorregador esquematizado ao
e a balança situam-se dentro da cabine de um lado e desce sem sofrer a ação de atritos ou da
elevador que tem, em relação à Terra, uma resistência do ar.
aceleração vertical de módulo 1,0 m/s2. Adotando
|g| = 10m/s2, calcule:
a) a indicação da balança no caso de o elevador
estar acelerado para cima;
b) a indicação da balança no caso de o elevador
estar acelerado para baixo.
Sabendo-se que no local a aceleração da
12. Em determinado parque de diversões, o elevador
que despenca verticalmente em queda livre é gravidade tem intensidade 10,0m/s2, responda:

AFA – EFOMM – EEAR| FÍSICA


a grande atração. Rafael, um garoto de massa a) Qual o módulo da aceleração adquirida pelo
igual a 70kg, encara o desafio e, sem se intimidar garoto? O valor calculado depende de sua
com os comentários de seus colegas, embarca no massa?
brinquedo, que começa a subir a partir do repouso.
b) Qual o intervalo de tempo gasto pelo garoto
Durante a ascensão vertical do elevador, são no percurso de A até B?
verificadas três etapas: c) Com que velocidade ele atinge o ponto B?
I. movimento uniformemente acelerado com
aceleração de módulo 1,0m/s2.
15. Dois blocos (1) e (2) de pesos respectivamente
II. movimento uniforme.
iguais a 30kgf e 10kgf estão em equilíbrio,
III. movimento uniformemente retardado com
aceleração de módulo 1,0m/s2. conforme mostra a figura abaixo:
Depois de alguns segundos estacionado no ponto
mais alto da torre, de onde Rafael acena triunfante
para o grupo de amigos, o elevador é destravado,
passando a cair com aceleração praticamente
igual à da gravidade (10m/s2). Pede-se calcular o
peso aparente de Rafael:
a) nas etapas I, II e III.
b) durante a queda livre. Quais as indicações dos dinamômetros D1 e D2,
graduados em kgf?
13. No plano inclinado representado ao lado, o bloco
encontra-se impedido de se movimentar devido
ao calço no qual está apoiado. Os atritos são
16. N a s i t u a ç ã o e s q u e m a t i z a d a n a f i g u r a ,
desprezíveis, a massa do bloco vale 5,0kg e
desprezam-se os atritos e a influência do ar. As
g = 10m/s2.
massas de A e B valem, respectivamente, 3,0kg
e 2,0kg. Sabendo-se que as forças F1 e F2 são
paralelas ao plano horizontal de apoio e que
|F1| = 40 N e |F2| = 10 N, pode-se afirmar que a
intensidade da força que B aplica em A vale:

a) 10N
b) 12N
a) Copie a figura esquematizando todas as forças
c) 18N
que agem no bloco.
b) Calcule as intensidades das forças com as quais d) 22N
o bloco comprime o calço e o plano de apoio. e) 26N
15 OS.:0424/13-Juliana
Prof. Rodrigo Lins

17. Na situação do esquema abaixo, não há atrito entre


os blocos e o plano horizontal, a influência do ar
é desprezível e as massas de A e de B valem,
respectivamente, 2,0kg e 8,0kg:

Sabe-se que o fio leve e inextensível que une


A com B suporta, sem romper-se, uma tração
máxima de 32N. Calcule a maior intensidade
admissível à força F, horizontal, para que o fio não
se rompa.

a) Qual a intensidade da aceleração do sistema?


b) Qual a intensidade da força de contato entre
AFA – EFOMM – EEAR| FÍSICA

os elos 4 e 5?
18. Na montagem esquematizada na figura, os blocos
A, B e C têm massas iguais a 2,0kg e a força
F, paralela ao plano horizontal de apoio, tem
21. Depois de regar o jardim de sua casa, José
intensidade 12N.
Raimundo enrolou cuidadosamente os 10m da
mangueira flexível utilizada na operação, deixando
um arremate de 60cm emergido do centro do

Desprezando todas as forças resistentes, calcule: rolo, conforme ilustra a figura. Querendo guardar

a) o módulo da aceleração do sistema. o acessório em uma prateleira elevada, o rapaz


b) as intensidades das forças de tração puxou o rolo para cima, exercendo, por alguns
estabelecidas nos fios ideais (1) e (2). instantes, uma força vertical F de intensidade
30,0N na extremidade do arremate.
19. Na figura, os blocos A, B e C têm massas
respectivamente iguais a 3M, 2M e M; o fio e a
polia são ideais. Os atritos são desprezíveis e a
aceleração da gravidade tem intensidade g.

Admitindo os blocos em movimento sob a ação


da gravidade, calcule as intensidades da força de Sabendo que a densidade linear da mangueira
tração no fio (T) e da força de contato trocada por (massa por unidade de comprimento) é igual a
B e C (F). 250g/m e que |g| = 10,0m/s2, calcule, durante o
20. Na figura, estão representadas uma caixa, de breve intervalo de tempo de atuação da força F:
massa igual a 4,7kg, e uma corrente constituída de
a) o módulo da aceleração adquirida pela
dez elos iguais, com massa de 50g cada um. Um
mangueira.
homem aplica no elo 1 uma força vertical dirigida
para cima, de intensidade 78N, e o sistema adquire b) a intensidade da força de tração em uma seção
aceleração. Admitindo |g| = 10m/s2 e desprezando S do arremate situada 20cm abaixo da mão
todos os atritos, responda: de José Raimundo.
OS.:0424/13-Juliana
16
Prof. Rodrigo Lins

22. Considere a montagem da figura abaixo: b) o intervalo de tempo decorrido desde o instante
em que a laranja é largada até o instante do
seu choque com o piso (a laranja é largada
de uma altura de 1,5m em relação ao piso do
elevador). Despreze o efeito do ar.

25. No arranjo experimental esquematizado na figura,


o fio e a polia são ideais, despreza-se o atrito
Os blocos A e B têm massas m A = 8,0kg e
entre o bloco A e o plano inclinado e adota-se
mB = 2,0kg; os fios, as roldanas e o dinamômetro
são ideais e despreza-se o efeito do ar. Adotando |g| = 10m/s2. Não levando em conta a influência
|g| = 10m/s2, determine: do ar, calcule:
a) o módulo da aceleração do sistema.

AFA – EFOMM – EEAR| FÍSICA


b) a indicação do dinamômetro (graduado em
newtons).

23. Na montagem experimental abaixo, os blocos


A, B e C têm massas mA = 5,0kg, mB = 3,0kg
e m C = 2,0kg. Desprezam-se os atritos e a
resistência do ar. Os fios e as polias são ideais e
adota-se |g| = 10m/s2.

Massa de A: 6,0kg
Massa de B: 4,0kg

No fio que liga A com B, está intercalada uma mola a) o módulo da aceleração dos blocos.
leve, de constante elástica 3,5 . 103N/m. Com o b) a intensidade da força de tração no fio.
sistema em movimento, calcule, em centímetros, c) a intensidade da força resultante que o fio
a deformação da mola. aplica na polia.

24. No esquema da figura, o garoto tem apoiada na 26. No arranjo experimental da figura, a caixa A é
palma de sua mão uma laranja de massa 100g. O acelerada para baixo com 2,0m/s2. As polias
elevador sobe aceleradamente, com aceleração
e o fio têm massas desprezíveis e adota- se
de módulo 2,0m/s2. Em dado instante, o garoto
|g| = 10m/s2.
larga a laranja, que se choca com o piso.

Supondo que a massa da caixa B seja de 80kg e


Supondo | g | = 10 m/s2, calcule: ignorando a influência do ar no sistema, determine:
a) a intensidade da força (em newtons) aplicada a) o módulo da aceleração de subida da caixa B.
pela laranja na mão do garoto enquanto em
b) a intensidade da força de tração no fio.
contato com ela.
c) a massa da caixa A.
17 OS.:0424/13-Juliana
Prof. Rodrigo Lins

27. Na figura, o sistema está sujeito à ação da 30. No esquema abaixo, o homem (massa de 80kg) é
resultante externa F, paralela ao plano horizontal acelerado verticalmente para cima juntamente com
sobre o qual o carrinho está apoiado. Todos os a plataforma horizontal (massa de 20kg) sobre a
atritos são irrelevantes e as inércias do fio e da qual está apoiado.
polia são desprezíveis. As massas dos corpos A, B
e C valem, respectivamente, 2,0kg, 1,0kg e 5,0kg
e, no local, o módulo da aceleração da gravidade
é 10m/s2.
AFA – EFOMM – EEAR| FÍSICA

Isso é possível porque ele puxa


Supondo que A esteja apenas encostado em C,
verticalmente para baixo a corda que passa
determine a intensidade de F de modo que A e B
pela polia fixa. A aceleração do conjunto
não se movimentem em relação ao carrinho C.
homem-plataforma tem módulo 5,0m/s2 e adota-se
|g| = 10m/s2. Considerando ideais a corda e a polia
28. (FEI-SP) Os blocos representados na figura abaixo e desprezando a influência do ar, calcule:
possuem, respectivamente, massas m1 = 2,0kg e a) a intensidade da força com que o homem puxa
m2 = 4,0kg; a mola AB possui massa desprezível e a corda;
constante elástica K = 50N/m. Não há atrito entre b) a intensidade da força de contato trocada entre
os dois blocos nem entre o bloco maior e o plano o homem e a plataforma.
horizontal.
31. O sistema indicado está em repouso devido à força
de atrito entre o bloco de massa de 10kg e o plano
horizontal de apoio. Os fios e as polias são ideais
e adota-se g = 10m/s2.

Aplicando ao conjunto a força F constante e


horizontal, verifica-se que a mola experimenta
uma deformação de 20cm. Qual a aceleração do
conjunto e a intensidade da força F?

29. A figura esquematiza dois blocos A e B de massas


respectivamente iguais a 6,0kg e 3,0kg em a) Qual o sentido da força de atrito no bloco de
movimento sobre o solo plano e horizontal. O bloco massa de 10kg, para a esquerda ou para a
B está simplesmente apoiado em uma reentrância direita?
existente no bloco A, não havendo atrito entre B b) Qual a intensidade dessa força?
e A.
32. Sobre um piso horizontal, repousa uma caixa
de massa 2,0 . 102kg. Um homem a empurra,
aplicando-lhe uma força paralela ao piso, conforme
sugere o esquema abaixo:

Admitindo que a intensidade da força horizontal F


que acelera o conjunto é 120N e que |g| = 10m/s2:
a) faça um esquema representando as forças que
agem no bloco A.
b) calcule a intensidade da força de contato que
A exerce em B.

OS.:0424/13-Juliana
18
Prof. Rodrigo Lins

O coeficiente de atrito estático entre a caixa e o 36. Na situação da figura, os blocos A e B têm
piso é 0,10 e, no local, g = 10m/s2. Determine: massas mA = 4,0kg e mB = 6,0kg. A aceleração
a) a intensidade da força com que o homem deve da gravidade no local tem módulo 10m/s2, o atrito
empurrar a caixa para colocá-la na iminência entre A e o plano horizontal de apoio é desprezível
de movimento; e o coeficiente de atrito estático entre B e A vale
b) a intensidade da força de atrito que se exerce sobre μe = 0,50.
a caixa quando o homem a empurra com 50N.

33. Na figura abaixo, um homem está empurrando


um fogão de massa 40kg, aplicando sobre ele
uma força F, paralela ao solo plano e horizontal.
O coeficiente de atrito estático entre o fogão
e o solo é igual a 0,75 e, no local, adota-se
g = 10m/s 2 . Supondo que o fogão está na
iminência de escorregar, calcule: Desprezando-se o efeito do ar, qual a máxima

AFA – EFOMM – EEAR| FÍSICA


a) a intensidade de F; intensidade da força F, paralela ao plano, de modo
b) a intensidade da força C de contato que o que B não se movimente em relação a A?
fogão recebe do solo.

34. Na situação da figura, o bloco B e o prato P pesam, 37. Considere duas caixas, A e B, de massas
respectivamente, 80 N e 1,0 N. O coeficiente de atrito respectivamente iguais a 10kg e 40kg, apoiadas
estático entre B e o plano horizontal de apoio vale sobre a carroceria de um caminhão que trafega
0,10 e desprezam-se os pesos dos fios e o atrito no em uma estrada reta, plana e horizontal. No local,
eixo da polia. No local, |g| = 10m/s2. Dispõe-se de a influência do ar é desprezível. Os coeficientes
20 blocos iguais, de 100g de massa cada um, que de atrito estático entre A e B e a carroceria valem
podem ser colocados sobre o prato P. μA = 0,35 e μB = 0,30 e, no local, g = 10m/s2.

Para que nenhuma das caixas escorregue, a


maior aceleração (ou desaceleração) permitida
ao caminhão tem intensidade igual a:
a) Colocando-se dois blocos sobre P, qual a a) 3,5m/s2;
intensidade da força de atrito exercida em B? b) 3,0m/s2;
b) Qual o número de blocos que deve ser c) 2,5m/s2;
colocado sobre P, para que B fique na d) 2,0m/s2;
iminência de se movimentar?
e) 1,5m/s2.

35. Sobre um plano inclinado, de ângulo θ variável,


38. Na figura, uma caixa de peso igual a 30kgf é
apoia-se uma caixa de pequenas dimensões,
mantida em equilíbrio, na iminência de deslizar,
conforme sugere o esquema abaixo. Sabendo-se
comprimida contra uma parede vertical por uma
que o coeficiente de atrito estático entre a caixa e
força horizontal F.
o plano de apoio vale 1,0, qual o máximo valor de
para que a caixa ainda permaneça em repouso?

19 OS.:0424/13-Juliana
Prof. Rodrigo Lins

Sabendo que o coeficiente de atrito estático entre O trecho BC é plano, reto e horizontal e oferece
a caixa e a parede é igual a 0,75, determine, em aos esquis um coeficiente de atrito cinético de valor
kgf:
0,20.
a) a intensidade de F ;
b) a intensidade da força de contato que a parede Admitindo desprezível a influência do ar e
aplica na caixa. adotando g = 10m/s2, determine:

a) a intensidade da aceleração de retardamento


39. Na situação esquematizada abaixo, um bloco de do esquiador no trecho BC;
peso igual a 40N está inicialmente em repouso
b) a distância percorrida por ele de B até C e o
sobre uma mesa horizontal. Os coeficientes de
intervalo de tempo gasto nesse percurso.
atrito estático e dinâmico entre a base do bloco
e a superfície da mesa valem, respectivamente,
0,30 e 0,25.
42. Os blocos A e B da figura abaixo têm massas
AFA – EFOMM – EEAR| FÍSICA

Admita que seja aplicada no bloco uma força


respectivamente iguais a 2,0kg e 3,0kg e estão
horizontal F.
sendo acelerados horizontalmente sob a ação de
uma força F de intensidade de 50N, paralela ao
plano do movimento. Sabendo que o coeficiente
de atrito de escorregamento entre os blocos e o
Adotando g = 10m/s2, indique os valores que plano de apoio vale μ = 0,60, que g = 10m/s2 e
preenchem as lacunas da tabela ao lado com as que o efeito do ar é desprezível, calcule:
intensidades da força de atrito e da aceleração do
bloco correspondentes às magnitudes definidas
para a força F.

F(N) Fat(N) a(m/s2)


10
12
30 a) o módulo da aceleração do sistema;
b) a intensidade da força de interação trocada
entre os blocos na região de contato.
40. Uma caixa de fósforos é lançada sobre uma mesa
horizontal com velocidade de 2,0m/s, parando
depois de percorrer 2,0m. No local do experimento,
43. (Unesp-SP) A figura ilusta um bloco A, de massa
a influência do ar é desprezível. Adotando para
m A = 2,0kg, atado a um bloco B, de massa
o campo gravitacional módulo igual a 10m/s2,
determine o coeficiente de atrito cinético entre a mB = 1,0kg, por um fio inextensível de massa
caixa e a mesa. desprezível. O coeficiente de atrito cinético
entre cada bloco e a mesa é μc. Uma força de

41. Na figura, o esquiador parte do repouso do ponto intensidade F = 18,0N é aplicada ao bloco B,
A, passa por B com velocidade de 20m/s e para fazendo com que os dois blocos se desloquem
no ponto C: com velocidade constante.

Considerando-se g = 10,0m/s2, calcule:

a) o coeficiente de atrito μc;


b) a intensidade da tração T no fio.
OS.:0424/13-Juliana
20
Prof. Rodrigo Lins

44. O corpo A, de 5,0kg de massa, está apoiado em um 47. No esquema abaixo, representa-se um livro
plano horizontal, preso a uma corda que passa por inicialmente em repouso sobre uma mesa
uma roldana de massa e atrito desprezíveis e que
horizontal, sendo empurrado horizontalmente por
sustenta em sua extremidade o corpo B, de 3,0kg
de massa. Nessas condições, o sistema apresenta um homem; F é a força que o homem aplica no
movimento uniforme. Adotando g = 10m/s2 e livro e Fat é a força de atrito exercida pela mesa
desprezando a influência do ar, determine: sobre o livro. Representa-se, também, como varia
a intensidade de Fat em função da intensidade
de F. No local, a influência do ar é desprezível e
adota-se |g| = 10m/s2.

AFA – EFOMM – EEAR| FÍSICA


a) o coeficiente de atrito cinético entre o corpo A
e o plano de apoio;
b) a intensidade da aceleração do sistema se
colocarmos sobre o corpo B uma massa de
2,0kg.
Com base no gráfico e nos demais dados,
45. (ESPCEX-SP – mod.) A figura abaixo representa
determine:
um automóvel em movimento retilíneo e acelerado
da esquerda para a direita. Os vetores desenhados a) os coeficientes de atrito estático e cinético
junto às rodas representam os sentidos das forças entre o livro e a mesa;
de atrito exercidas pelo chão sobre as rodas.
b) o módulo da aceleração do livro quando
Sendo assim, pode-se afirmar que o automóvel:
F = 18N.

48. Considere o esquema abaixo, em que se


representa um bloco de 1,0kg de massa apoiado
sobre um plano horizontal. O coeficiente de
a) tem tração apenas nas rodas traseiras. atrito de arrastamento entre a base do bloco e a
b) tem tração nas quatro rodas. superfície de apoio vale 0,25 e a aceleração da
c) tem tração apenas nas rodas dianteiras. gravidade, no local, tem módulo 10m/s2.
d) move-se em ponto morto, isto é, sem que
nenhuma das rodas seja tracionada.
e) está em alta velocidade.

46. Um bloco de 2,0kg de massa repousa sobre um plano


horizontal quando lhe é aplicada uma força F, paralela
ao plano, conforme representa a figura abaixo:
A força F, cuja intensidade é de 10N, forma com
a direção horizontal um ângulo θ constante, tal
que sen θ = 0,60 e cos θ = 0,80. Desprezando a
influência do ar, aponte a alternativa que traz o
Os coeficientes de atrito estático e cinético entre o valor correto da aceleração do bloco:
bloco e o plano de apoio valem, respectivamente,
0,50 e 0,40 e, no local, a aceleração da gravidade a) 7,0m/s2
tem módulo 10 m/s2. Calcule: b) 5,5m/s2
a) a intensidade da força de atrito recebida pelo
c) 4,0m/s2
bloco quando |F| = 9,0N;
b) o módulo da aceleração do bloco quando d) 2,5m/s2
|F| = 16N. Despreze o efeito do ar. e) 1,5m/s2
21 OS.:0424/13-Juliana
Prof. Rodrigo Lins

49. (Efomm-RJ) Os blocos A e B representados 52. A situação representada na figura refere-se a um


na figura possuem massas de 3,0kg e 2,0kg bloco que, abandonado em repouso no ponto
respectivamente. A superfície horizontal onde eles A, desce o plano inclinado com aceleração de
se deslocam apresenta um coeficiente de atrito
2,0m/s 2, indo atingir o ponto B. Sabendo-se
cinético igual a 0,30. F1 e F2 são forças horizontais
que, no local, |g| = 10m/s2 e a influência do ar
que atuam nos blocos.
é desprezível, pede-se calcular o coeficiente de
atrito cinético entre o bloco e o plano de apoio.

Adotando g = 10m/s2 e desprezando o efeito do


ar, determine:
a) o módulo da aceleração do sistema;
b) a intensidade da força de contato entre A e B.
AFA – EFOMM – EEAR| FÍSICA

53. Na situação esquematizada na figura, o fio e a polia


50. (Vunesp-SP) Dois blocos, A e B, ambos de massa
são ideais; despreza-se o efeito do ar e adota-se
m, estão ligados por um fio leve e flexível, que
passa por uma polia de massa desprezível, que g = 10m/s2. sen θ = 0,60 e cos θ = 0,80.
gira sem atrito. O bloco A está apoiado sobre um
carrinho de massa 4m, que pode se deslocar sobre
a superfície horizontal sem encontrar qualquer
resistência. A figura mostra a situação descrita.

Sabendo que os blocos A e B têm massas iguais


a 5,0kg e que os coeficientes de atrito estático
e cinético entre B e o plano de apoio valem,
respectivamente, 0,45 e 0,40, determine:
Quando o conjunto é liberado, B desce e A se
a) o módulo da aceleração dos blocos;
desloca com atrito constante sobre o carrinho,
acelerando-o. Sabendo que o coeficiente de atrito b) a intensidade da força de tração no fio.
dinâmico entre A e o carrinho vale 0,20 e fazendo
g = 10 m/s2, determine:
54. (Vunesp-SP) Na figura, o bloco I repousa sobre
a) o módulo da aceleração do carrinho; o bloco II, sendo que I está preso por uma corda
b) o módulo da aceleração do sistema constituído
a uma parede. mI = 3,0kg e mII = 6,0kg. O
por A e B.
coeficiente de atrito cinético entre I e II é 0,10 e
51. Na figura abaixo, a superfície S é horizontal, a entre II e o plano é 0,20. Qual deve ser a força
intensidade de F é 40N, o coeficiente de atrito de F que, aplicada em II, desloca esse bloco com
arrastamento entre o bloco A e a superfície S vale aceleração de 2,0m/s2? (g = 10m/s2)
0,50 e g = 10m/s2.

Sob a ação da força F, o sistema é acelerado


horizontalmente e, nessas condições, o bloco B
apresenta-se na iminência de escorregar em relação a) 15N
ao bloco A. Desprezando a influência do ar: b) 27N
a) determine o módulo da aceleração do sistema; c) 30N
b) calcule o coeficiente do atrito estático entre os d) 33N
blocos A e B. e) 40N
OS.:0424/13-Juliana
22
Prof. Rodrigo Lins

Considere a situação seguinte, referente aos 59. Analise as proposições seguintes:


exercícios de 55 a 59.
I. Ao longo da trajetória, a componente tangencial
da força resultante que age no carrinho tem
No esquema abaixo aparece, no ponto P, um carrinho intensidade variável.
de massa 2,0kg, que percorre a trajetória indicada II. Ao longo da trajetória, a componente tangencial
da esquerda para a direita. A aceleração escalar do da força resultante que age no carrinho é
carrinho é constante e seu módulo vale 0,50m/s2. As constante.
setas enumeradas de I a V representam vetores que
III. Ao longo da trajetória, a velocidade vetorial do
podem estar relacionados com a situação proposta.
carrinho tem intensidade variável.
IV. Quem provoca as variações do módulo da
velocidade do carrinho ao longo da trajetória
é a componente tangencial da força resultante
que age sobre ele.

AFA – EFOMM – EEAR| FÍSICA


Responda mediante o código:
a) Todas são corretas.
b) Todas são incorretas.
55. A velocidade vetorial do carrinho em P é mais bem c) Somente I e II são corretas.
representada pelo vetor: d) Somente III e IV são corretas.
a) I e) Somente II, III e IV são corretas.
b) II
c) III 60. Considere um carro de massa 1,0 . 10 3 kg
d) IV percorrendo, com velocidade escalar constante,
e) V uma curva circular de 125m de raio, contida em
um plano horizontal. Sabendo que a força de
56. Se o movimento for acelerado, a componente atrito responsável pela manutenção do carro na
tangencial da força resultante que age no carrinho curva tem intensidade 5,0kN, determine o valor da
em P será mais bem representada pelo vetor: velocidade do carro. Responda em km/h.

a) I
b) II 61. Uma partícula percorre certa trajetória curva e
c) III plana, como a representada nos esquemas a
d) IV seguir.
e) V
Em P, a força resultante que age sobre ela é F e
sua velocidade vetorial é v:
57. Se o movimento for retardado, a componente
tangencial da força resultante que age no carrinho
em P será mais bem representada pelo vetor:
a) I
b) II
c) III
d) IV
e) V
Nos casos I, II e III, a partícula está dotada de um
dos três movimentos citados abaixo:
58. A intensidade da componente tangencial da força
A — movimento uniforme;
resultante que age no carrinho em P vale:
B — movimento acelerado;
a) zero C — movimento retardado.
b) 2,0N
c) 1,0N A alternativa que traz as associações corretas é:
d) 0,50N a) I – A; II – B; III – C. d) I – B; II – C; III – A.
e) 0,25N
b) I – C; II – B; III – A. e) I – A; II – C; III – B.
c) I – B; II – A; III – C.
23 OS.:0424/13-Juliana
Prof. Rodrigo Lins

62. Uma pista é constituída por três trechos: dois permanece em repouso quando a velocidade da
retilíneos, AB e CD, e um circular, BC, conforme esfera é v = 10m/s.
representa a vista aérea abaixo.

Sendo g = 10m/s2, calcule o raio da trajetória


da esfera, observando a condição de o bloco
permanecer em repouso.
AFA – EFOMM – EEAR| FÍSICA

Admita que um carro de massa m percorra a pista


com velocidade de intensidade constante igual a 65. A figura representa duas esferas iguais, E1 e E2,
v. que, ligadas a fios inextensíveis e de massas
Sendo R o raio do trecho BC, analise as desprezíveis, descrevem movimento circular e
proposições a seguir: uniforme sobre uma mesa horizontal perfeitamente
(01) No trecho AB, a força resultante sobre o lisa:
carro é nula.
(02) No trecho CD, a força resultante sobre o
carro é não-nula.
(04) Em qualquer ponto do trecho BC, a força
resultante sobre o carro é dirigida para
o ponto O e sua intensidade é dada por
m.v 2 .
R Desprezando o efeito do ar e supondo que E1 e
(08) No trecho BC, a força resultante sobre o E2 se mantenham sempre alinhadas com o centro,
carro é constante.
aponte a alternativa que traz o valor correto da
(16) De A para D, a variação da velocidade
relação T1/T2, respectivamente das forças de
vetorial do carro tem intensidade v. 2 .
tração nos fios (1) e (2):
Dê como resposta a soma dos números associados
às proposições corretas. a) 2
b) 3/2

63. Um bloco de massa 4,0kg descreve movimento c) 1


circular e uniforme sobre uma mesa horizontal d) 2/3
perfeitamente polida. Um fio ideal, de 1,0m de e) 1/2
comprimento, prende-o a um prego C, conforme
ilustra o esquema:
66. (Unesp-SP) Numa calçada de uma rua plana e
horizontal, um patinador vira em uma esquina,
descrevendo um arco de circunferência de 3,0m
de raio. Admitindo-se g = 10m/s2 e sabendo-se
que o coeficiente de atrito estático entre as rodas
Se a força de tração no fio tem intensidade do patim e a calçada é μe = 0,30, a máxima
1,0 · 102N, qual a velocidade angular do bloco,
velocidade com que o patinador pode realizar a
em rad/s?
manobra sem derrapar é de:
a) 1,0m/s.
64. Na figura a seguir, uma esfera de massa m = 2,0kg b) 2,0m/s.
descreve sobre a mesa plana, lisa e horizontal
c) 3,0m/s.
um movimento circular. A esfera está ligada por
um fio ideal a um bloco de massa M = 10kg, que d) 5,0m/s.
e) 9,0m/s.
OS.:0424/13-Juliana
24
Prof. Rodrigo Lins

67. Um carro deverá fazer uma curva circular, contida A velocidade escalar do carrinho vale 8,0m/s, sua
em um plano horizontal, com velocidade de trajetória pertence a um plano vertical e adota-se
intensidade constante igual a 108km/h. Se o raio |g| = 10m/s2. Supondo que os pontos A e B sejam,
da curva é R = 300m e g = 10m/s2, o coeficiente respectivamente, o mais alto e o mais baixo do
de atrito estático entre os pneus do carro e a pista trilho, determine a intensidade da força que o trilho
(μ) que permite que o veículo faça a curva sem exerce no carrinho:
derrapar: a) no ponto A.
a) é με ≥ 0,35 b) no ponto B.
b) é με ≥ 0,30
c) é με ≥ 0,25
70. (UFRJ) A figura representa uma roda-gigante que
d) é με ≥ 0,20
gira com velocidade angular constante em torno de
e) está indeterminado, pois não foi dada a massa um eixo horizontal fixo que passa por seu centro C.
do carro.

AFA – EFOMM – EEAR| FÍSICA


68. Um estudante, indo para a faculdade em seu carro,
desloca-se num plano horizontal, no qual descreve
uma trajetória curvilínea de 48m de raio, com uma
velocidade constante em módulo. Entre os pneus
e a pista, o coeficiente de atrito estático é de 0,30.

Numa das cadeiras, há um passageiro sentado


sobre uma balança de mola (dinamômetro), cuja
indicação varia de acordo com a posição do
passageiro. No ponto mais alto da trajetória, o
dinamômetro indica 234N e, no ponto mais baixo,
indica 954N. Calcule:
Considerando-se a figura, a aceleração da a) o peso da pessoa;
gravidade no local, com módulo de 10m/s2, e a b) a intensidade da força resultante na pessoa.
massa do carro de 1,2t, faça o que se pede:
a) Caso o estudante resolva imprimir uma 71. O pêndulo da figura oscila em condições ideais,
velocidade de módulo 60km/h ao carro, ele invertendo sucessivamente o sentido do seu
conseguirá fazer a curva? Justifique. movimento nos pontos A e C:
b) A velocidade escalar máxima possível, para
que o carro possa fazer a curva, sem derrapar,
irá se alterar se diminuirmos sua massa?
Explique.

69. Na figura abaixo, um carrinho de massa 1,0kg


descreve movimento circular e uniforme ao longo
de um trilho envergado em forma de circunferência
de 2,0m de raio:
A esfera tem massa 1,0kg e o comprimento do fio,
leve e inextensível, vale 2,0m. Sabendo que no
ponto B (mais baixo da trajetória) a esfera tem
velocidade de módulo 2,0m/s e que |g| = 10m/s2,
determine:
a) a intensidade da força resultante sobre a
esfera quando ela passa pelo ponto B.
b) a intensidade da força que traciona o fio
quando a esfera passa pelo ponto B.

25 OS.:0424/13-Juliana
Prof. Rodrigo Lins

72. Uma moto percorre um morro, conforme ilustra a 74. (Unicamp-SP) Uma atração muito popular nos
figura abaixo. Visto em corte, esse morro pode ser circos é o “Globo da Morte”, que consiste em
comparado a um arco de circunferência de raio R, uma gaiola de forma esférica no interior da
contido em um plano vertical. Observe: qual se movimenta uma pessoa pilotando uma
motocicleta.

Considere um globo de raio R = 3,6m.

Ao passar no ponto A, o mais alto do morro, a moto


recebe da pista uma força de reação normal 25%
AFA – EFOMM – EEAR| FÍSICA

menor que aquela que receberia se estivesse em


repouso nesse ponto. Se no local a aceleração da
gravidade vale g, qual será o módulo da velocidade
da moto no ponto A? a) Reproduza a figura, fazendo um diagrama
das forças que atuam sobre a motocicleta
73. A ilustração abaixo representa um globo da morte,
nos pontos A, B, C e D sem incluir as forças
dentro do qual um motociclista realiza evoluções
de atrito. Para efeitos práticos, considere o
circulares contidas em um plano vertical.
conjunto piloto + motocicleta como sendo um
O raio da circunferência descrita pelo conjunto ponto material.
motopiloto é igual ao do globo e vale R. b) Qual a velocidade mínima que a motocicleta
deve ter no ponto C para não perder o contato
com o interior do globo? Adote |g| = 10m/s2.

75. Uma partícula de massa 3,0kg parte do repouso


no instante t0 = 0, adquirindo movimento circular
uniformemente acelerado. Sua aceleração escalar
é de 4,0m/s2 e o raio da circunferência suporte do
movimento vale 3,0m. Para o instante t1 = 1,0s,
calcule a intensidade da força resultante sobre a
O ponto A é o mais alto da trajetória e por lá o
partícula.
conjunto moto-piloto, que tem massa M, passa
com a mínima velocidade admissível para não
perder o contato com a superfície esférica.
76. Na situação esquematizada na figura, a mesa é
Supondo que a aceleração da gravidade tenha
plana, horizontal e perfeitamente polida. A mola
módulo g, analise as proposições a seguir:
tem massa desprezível, constante elástica igual
(01) No ponto A, a força vertical trocada pelo
a 2,0 . 102 N/m e comprimento natural (sem
conjunto moto-piloto e o globo é nula.
deformação) de 80cm.
(02) No ponto A, a força resultante no conjunto
moto-piloto tem intensidade M g.
(04) No ponto A, o peso do conjunto
moto-piloto desempenha a função de
resultante centrípeta.
(08) No ponto A, a velocidade do conjunto
moto-piloto tem módulo R.g .
(16) Se a massa do conjunto moto-piloto fosse
2M, sua velocidade no ponto A teria módulo
2.R.g .
Se a esfera (massa de 2,0kg) descreve movimento
Dê como resposta a soma dos números associados
circular e uniforme, qual o módulo da sua
às proposições corretas.
velocidade tangencial?
OS.:0424/13-Juliana
26
Prof. Rodrigo Lins

77. O esquema abaixo representa um disco horizontal


que, acoplado rigidamente a um eixo vertical, gira
uniformemente sem sofrer resistência do ar:

a) calcule o período de revolução do pêndulo;

b) discuta, justificando, se o período calculado


no item anterior seria modificado se o pêndulo

AFA – EFOMM – EEAR| FÍSICA


Sobre o disco, estão apoiados dois blocos, A e B,
fosse levado para um outro local, de aceleração
constituídos de materiais diferentes, que distam
do eixo 40cm e 20cm respectivamente. Sabendo da gravidade igual a g/4.

que, nas condições do problema, os blocos estão


na iminência de deslizar, obtenha:
80. Em alguns parques de diversões, existe um
a) a relação vA/vB das velocidades lineares de A
e de B em relação ao eixo; brinquedo chamado rotor, que consiste em um

b) a relação μA/μB dos coeficientes de atrito cilindro oco, de eixo vertical, dentro do qual é
estático entre os blocos A e B e o disco. introduzida uma pessoa:

78. Na figura abaixo, representa-se um pêndulo


fixo em O, oscilando num plano vertical. No
local, despreza-se a influência do ar e adota-se
g = 10m/s2. A esfera tem massa de 3,0kg e o fio
é leve e inextensível, apresentando comprimento
de 1,5m.

Se, na posição A, o fio forma com a direção vertical


um ângulo de 53° e a esfera tem velocidade igual a
2,0m/s, determine a intensidade da força de tração
no fio. Dados: sen 53° = 0,80; cos 53° = 0,60.

De início, a pessoa apoia-se sobre um suporte,


que é retirado automaticamente quando o rotor
gira com uma velocidade adequada. Admita
que o coeficiente de atrito estático entre o corpo
da pessoa e a parede interna do rotor valha
79. No esquema a seguir, representa-se um pêndulo
cônico operando em condições ideais. A esfera μ. Suponha que o módulo da aceleração da
pendular descreve movimento circular e uniforme, gravidade seja g e que o rotor tenha raio R.
num plano horizontal, de modo que o afastamento
Calcule a mínima velocidade angular do rotor, de
angular do fio em relação à vertical é θ . Sendo g o
módulo do campo gravitacional do local e r o raio modo que, com o suporte retirado, a pessoa não
da circunferência descrita pela esfera pendular: escorregue em relação à parede.
27 OS.:0424/13-Juliana
Prof. Rodrigo Lins

Gabarito dos Exercícios de Fixação


06. 5m/s2
01 02 03 04 05 06 07 08
d c b c d b e d 07. a) 2m/s2 b) 4N
09 10 11 12 13 14 15 16 mB mA mB
09. a) a = g b) T= g
c c d e e c a e mA + mB mA + mB
17 18 19 20 21 22 23 24
c * c d c c e c 10. a) 2m/s2 b) 24N c) 48N
25 26 27 28 29 30 31 32 11. a) 600N b) 540N
a a d d b d d d
12. a) 770N, 700N e 630N b) zero
33 34 35 36 37 38 39 40
13. b) 30N e 40N
a * a c e b d e
41 42 14. a) 5m/s2 b) 1s c) 5m/s
AFA – EFOMM – EEAR| FÍSICA

e d 15. 30kgf e 20kgf


17. 40N
18. a) a = g.tgα
18. a) 2m/s2 b) 8N e 4N
b) O módulo de a não depende de m, que foi
cancelada nos cálculos. g 3Mg Mg
=
19. a =; T ;=
FBC
2 2 2
20. a) 5m/s2 b) 75N
34. a) 6m/s2 b) 4,8 KN
21. a) 2m/s2 b) 29,4N
22. a) 6m/s2 b) 32N
23. 1cm
Gabarito dos Exercícios Propostos
24. a) 1,2N b) 0,5s
25. a) 7m/s2 b) 12N c) 12N
01 02 03 04 05 06 07 08
c c * * c * * c 26. a) 1m/s2 b) 440N c) 55kg
09 10 11 12 13 14 15 16 27. 160N
* * * * * * * d 28. 5m/s2 e 30N
17 18 19 20 21 22 23 24
29. b) 50N
* * * * * * * *
30. a) 750N b) 450N
25 26 27 28 29 30 31 32
* * * * * * * * 31. a) esquerda b) 20N
33 34 35 36 37 38 39 40 32. a) 200N b) 50N
* * * * b * * *
33. a) 300N b) 500N
41 42 43 44 45 46 47 48
34. a) 3N b) 7 bloquinhos
* * * * a * * a
49 50 51 52 53 54 55 56 35. 45º
* * * * * d a a 36. 50N
57 58 59 60 61 62 63 64 38. a) 40kgf b) 50kgf
e c d * c * * *
39.
65 66 67 68 69 70 71 72
d c b * * * * * F(N) Fat(N) a(m/s2)
73 74 75 76 77 78 79 80 10 10 0
* * * * * * * * 12 12 0
30 10 5,0
03. 16,5cm 40. 0,1
04. 5m/s2 41. a) 2m/s2 b) 100m; 10s

OS.:0424/13-Juliana
28
Prof. Rodrigo Lins

42. a) 4m/s2 b) 30N Trabalho


43. a) 0,6 b) 12N
44. a) 0,6 b) 2m/s2 1. Introdução
46. a) 9N b) 4m/s2 Realizar trabalho em Física implica a
transferência de energia de um sistema para outro e,
47. a) 0,2 e 0,16 b) 2m/s2
para que isso ocorra, são necessários uma força e um
49. a) 1m/s2 b) 18N deslocamento adequados.
50. a) 0,5m/s2 b) 4m/s2
2. Trabalho de uma força constante
51. a) 3m/s2 b) 0,3
Consideremos a figura abaixo, em que uma
52. 0,5 partícula é deslocada de A até B, ao longo da trajetória
53. a) 0,4m/s2 b) 48N indicada. O trabalho (τ) da força F no deslocamento
de A a B é a grandeza escalar dada por:
60. 90km/h

AFA – EFOMM – EEAR| FÍSICA


62. 21
63. 5 rad/s
64. 2m
68. a) não conseguirá
b) independe da massa m → →
τ = |F | |d | cos θ
69. a) 22N b) 42N
70. a) 594N b) 360N
No Sistema Internacional (SI), o trabalho é
71. a) 2N b) 12N medido em joule (J), em homenagem ao físico
1 inglês James Prescott Joule (1818-1889), que deu
72. v = gR importantes contribuições à Termodinâmica.
2
73. 15
NOTA:
74. b) 6m/s Sinais do trabalho
75. 20N O trabalho é uma grandeza algébrica, isto é,
admite valores positivos e negativos. Quem impõe o
76. 3m/s sinal do trabalho é o cos θ.
77. a) 2 b) 2
3. Casos particulares importantes
78. 26N
3.1. Quando F e d têm a mesma direção e o mesmo
r
79. a) T= 2π sentido.
g tgθ
b) O período ficaria multiplicado por 2, já que ele
é inversamente proporcional à raiz quadrada
da intensidade da aceleração da gravidade. τ=Fd

g Esse é o caso em que a força realiza seu trabalho


80. ωmn = máximo.
µR

3.2. Quando F e d têm a mesma direção e sentidos


opostos.
Referência Bibliográfica
I)DOCA, R.H.; BISCULA,G.J. e BOAS, N.V. Tópicos τ = –F d
de Física, v.1, Ed.18ª, Saraiva (2010).

II)JUNIOR,F.R.; FERRARO, N.G. e SOARES,P.A.T. Sempre que a força e o deslocamento forem


Fundamentos da Física, v.1, Ed 9ª. Moderna (2010). perpendiculares entre si, a força não realizará
trabalho.
29 OS.:0424/13-Juliana
Prof. Rodrigo Lins

3.2. Quando F e d são perpendiculares entre si. O módulo do trabalho de Fe é dado pela "área" A,
destacada no diagrama:
Sempre que a força e o deslocamento forem
perpendiculares entre si, a força não realizará trabalho
A força centrípeta nunca realiza trabalho; seu trabalho
é sempre nulo.

Levando em conta que τp pode ser motor (+) ou


τ=0 resistente (-), escrevemos:

K( ∆x)2
τFe
 =±
4. Cálculo gráfico do trabalho 2
AFA – EFOMM – EEAR| FÍSICA

Dado um diagrama do valor algébrico da força NOTA:


atuante em uma partícula em função da sua posição, Uma força é denominada conservativa quando
a "área" compreendida entre o gráfico e o eixo das seu trabalho, entre duas posições, independe da
posições expressa o valor algébrico do trabalho trajetória descrita pelo seu ponto de aplicação. Diante
da força. No entanto, a força considerada deve ser disso, temos que a força peso e a força elástica são
conservativas.
paralela ao deslocamento da partícula.
7. Potência média
Do ponto de vista geral, a potência de um
sistema consiste na rapidez com que ele realiza
suas atribuições. A potência é tanto maior quanto
menor é o intervalo de tempo utilizado na execução
de uma mesma tarefa. Nesse processo, define-se
A=τ
potência média (Potm) como o quociente da energia
transferida (∆E) pelo intervalo de tempo (∆t) em que
essa transferência ocorreu. Essa energia transferida
5. Trabalho da força peso equivale a um trabalho, assim, a potência média
também pode ser dada por:
O trabalho da força peso é independente da Unidades:
trajetória descrita pela partícula. τ
Pot m =
∆t

• (S.l) unid (Pot) = 1 J / s = watt (W)


• Um múltiplo muito usado do watt é o quilowatt
(kW): 1 KW 103W
• Embora não-pertencentes ao Sistema Internacional
(SI), são também muito empregadas as seguintes
τp = ±Ph = ±m g h unidades de potência:
1 cv = 735,5 W e 1 HP = 745,7W
O trabalho do peso é positivo na descida.
O trabalho do peso é negativo na subida.
8. Relação entre potência instantânea e velocidade
6. Trabalho da força elástica Em vários problemas de Mecânica, há interesse
Chamemos de força elástica (Fe) a força em se relacionar a potência com a velocidade.
Conhecendo, por exemplo, a intensidade da
aplicada pela mola na mão do operador. Essa força é
velocidade de um veículo, podemos determinar a
denominada força de restituição. potência útil fornecida pelo seu motor.

Fe = K∆x Pot = F v

OS.:0424/13-Juliana
30
Prof. Rodrigo Lins

9. Propriedade do gráfico da potência em função • Energia Eólica


do tempo O vento age nas pás dos rotores, fazendo-as
Dado um diagrama da potência em função do girar.
tempo, a "área" compreendida entre o gráfico e o eixo Esse movimento é transmitido aos eixos de
geradores, que disponibilizam em seus terminais
dos tempos expressa o valor algébrico do trabalho ou
tensão elétrica. A potência útil disponível em cada
dá energia transferida. ventoinha é sempre menor que a potência recebida
do vento, já que sempre ocorrem dissipações. Isso
indica que o rendimento de cada sistema captador é
menor que 100%.

A=τ

Energia

AFA – EFOMM – EEAR| FÍSICA


10. Rendimento
O rendimento (η) de um sistema físico qualquer 1. Introdução
é dado pelo quociente da potência útil (Potu) pela A energia desempenha um papel essencial em
potência recebida (Potr). todos os setores da vida, sendo a grandeza mais
importante da Física.
A energia é uma grandeza única, mas,
Pot u dependendo de como se manifesta, recebe diferentes
η= denominações:
Pot r
• energia térmica;
Sendo Potu a potência útil (utilizada no • energia luminosa;
movimento) e Potd a potência dissipada, temos que: • energia elétrica;
• energia química;
• energia mecânica;
Potr • energia atômica, dentre outras.
A energia total do Universo é constante, podendo
Pot u SISTEMA haver apenas transformações de uma modalidade em
η= FÍSICO Perdas
Pot r outras.
(hidrelétrica) Potd
Potu
2. Unidades de energia
As unidades de energia são as mesmas de
Potu = Potr – Potd trabalho. Recordando, vimos que, no SI:

Notas:
unid (energia) = unid (trabalho) = joule (J)
• O rendimento é adimensional (não tem unidades),
por ser definido pelo quociente de duas grandezas
Outras unidades de energia:
medidas nas mesmas unidades. É expresso
• Caloria (cal): utilizada nos fenômenos
geralmente em porcentagem, bastando, para isso, térmicos.
multiplicar o seu valor por 100%. 1 Cal = 4,19J
• O rendimento de um sistema físico real é sempre
inferior a 1 ou a 100%, pois, devido às dissipações • Quilowatt-hora (kWh): utilizada em geração
sempre existentes, a potência útil é sempre menor e distribuição de energia elétrica.
que a recebida.
1 kWh = 3,6.106J
• Dizemos que o rendimento de um sistema é
baixo quando a potência útil é bem menor que a • Elétron-volt (eV): utilizada nos estudos do
recebida. átomo.
• Dizemos que o rendimento de um sistema é
1 eV = 1,602.10–19J
alto quando a potência útil é pouco menor que a
recebida.
31 OS.:0424/13-Juliana
Prof. Rodrigo Lins

3. Energia cinética A variação de energia potencial de gravidade


Considere a figura seguinte, em que uma (∆Ep) é a diferença entre as energias potenciais final
partícula de massa m encontra-se em repouso no (Epf) e inicial (Epi)
ponto A do plano horizontal. Uma pessoa empurra a
partícula, aplicando-lhe a força F indicada, constante
e paralela ao plano de apoio. ∆EP= Epf – Epi

mv 2 4.2 Energia potencial elástica (Ee)


EC =
2 É a forma de energia que encontramos
armazenada em sistemas elásticos deformados.
• A energia cinética é uma grandeza relativa, pois é É o caso, por exemplo, de uma mola alongada ou
função da velocidade, que depende do referencial. comprimida ou de uma tira de borracha alongada.
Assim, uma única partícula pode ter, ao mesmo
tempo, energia cinética nula para um referencial
AFA – EFOMM – EEAR| FÍSICA

e não-nula para outro.


• A energia cinética (Ec) de uma partícula é
proporcional ao quadrado de sua velocidade
escalar (v). Graficamente, temos:

Por estar deformada, dizemos que a mola


está energizada, tendo armazenada em si energia
potencial elástica (Ee).
Observe que a energia cinética é exclusivamente
positiva.
K( ∆x)2
Ee =
4. Energia potencial 2
É uma forma de energia latente, isto está sempre
• A energia potencial elástica (Ee) é exclusivamente
prestes a se converter em energia cinética, na
mecânica há duas modalidades de energia potencial. positiva; é diretamente proporcional ao quadrado
• energia potencial de gravidade; da deformação (∆x). Assim, o gráfico Ee versus
• energia potencial elástica. ∆x é um arco de parábola, como representamos
abaixo.
4.1 Energia potencial de gravidade
A energia potencial de gravidade deve ser
definida em relação a um determinado plano horizontal
de referência (PHR), a partir do qual são medidas as
alturas.

EP=P h ou Ep = m g h ∆X < O (mola comprimida)


∆X > O (mola alongada)
Um mesmo corpo pode ter energia potencial de
gravidade positiva, nula ou negativa, dependendo do
PHR adotado. 5. Cálculo da energia mecânica
Calculamos a energia mecânica (EM) de um
sistema adicionando a energia cinética à energia
potencial, que pode ser de gravidade ou elástica:

Em= Ecinética + Epotencial

OS.:0424/13-Juliana
32
Prof. Rodrigo Lins

6. Sistema mecânico conservativo pela força F2 , que realiza um trabalho τ2; e na


Sistema mecânico conservativo é todo aquele terceira é puxada por uma força F3 , que realiza
em que as forças que realizam trabalho transformam um trabalho τ3.
exclusivamente energia potencial em energia cinética
e vice-versa. Supondo os comprimentos dos vetores da figura
Podemos dizer, então, que um sistema mecânico proporcionais às intensidades de F1, F2 e F3,
só é conservativo quando o trabalho é realizado aponte a alternativa correta.
exclusivamente por forças conservativas.
a) τ1 > τ2 > τ3
7. Princípio da conservação da energia mecânica b) τ1< τ2 < τ3
Num sistema mecânico conservativo, a energia c) τ1= τ2 = τ3
mecânica total é sempre constante. d) τ1 = τ2 = 0
e) τ1= τ2 < τ3

AFA – EFOMM – EEAR| FÍSICA


04. Considere um garoto de massa igual a 50kg em
Exercícios uma roda-gigante que opera com velocidade
angular constante de 0,50 rad/s. Supondo que a
distância entre o garoto e o eixo da roda-gigante
01. Na figura abaixo, embora puxe a carroça com
seja de 4,0m, calcule:
uma força horizontal de 1,0 . 103N, o burro não
consegue tirá-la do lugar devido ao entrave de a) a intensidade da força resultante no corpo do
uma pedra: garoto;
b) o trabalho realizado por essa força ao longo
de meia volta.

05. A intensidade da resultante das forças que agem


em uma partícula varia em função de sua posição
sobre o eixo Ox, conforme o gráfico a seguir:
Qual o trabalho da força do burro sobre a carroça?

02. Um homem empurra um carrinho ao longo de


uma estrada plana, comunicando a ele uma
força constante, paralela ao deslocamento, e
de intensidade 3,0 . 102N. Determine o trabalho
realizado pela força aplicada pelo homem sobre o
carrinho, considerando um deslocamento de 15m.

Calcule o trabalho da força para os deslocamentos:


03. No esquema da figura, uma mesma caixa é
a) de x1 =0 a x2 = 8,0m;
arrastada três vezes ao longo do plano horizontal,
deslocando-se do ponto A até o ponto B: b) de x2 = 8,0m a x3 = 12m;
c) de x1 = 0 a x3 = 12m.

06. Uma força constante F, horizontal, de intensidade


20N, atua durante 8,0s sobre um corpo de massa
4,0kg que estava em repouso apoiado em uma
superfície horizontal perfeitamente sem atrito.
Não se considera o efeito do ar. Qual o trabalho
Na primeira vez, é puxada pela força F1 , que
realizado pela força F no citado intervalo de
realiza um trabalho τ1; na segunda, é puxada
tempo?
33 OS.:0424/13-Juliana
Prof. Rodrigo Lins

07. Um carregador em um depósito empurra, sobre o 10. O esquema abaixo ilustra um homem que,
solo horizontal, uma caixa de massa 20kg, que puxando a corda verticalmente para baixo
com força constante, arrasta a caixa de peso
inicialmente estava em repouso. Para colocar
4,0 . 102N em movimento uniforme, ao longo do
a caixa em movimento, é necessária uma força plano inclinado:
horizontal de intensidade 30N. Uma vez iniciado
o deslizamento, são necessários 20N para manter
a caixa movendo-se com velocidade constante.
Considere g = 10m/s2.

a) Determine os coeficientes de atrito estático e


cinético entre a caixa e o solo.
Desprezando os atritos e a influência do ar e
b) Determine o trabalho realizado pelo carregador admitindo que a corda e a roldana sejam ideais,
ao arrastar a caixa por 5m. calcule o trabalho da força exercida pelo homem
AFA – EFOMM – EEAR| FÍSICA

ao provocar na caixa um deslocamento de 3,0m


c) Qual seria o trabalho realizado pelo carregador
na direção do plano inclinado.
se a força horizontal aplicada inicialmente
fosse de 20N? Justifique sua resposta. 11. O gráfico abaixo representa a variação do valor
algébrico das duas únicas forças que agem em
um corpo que se desloca sobre um eixo Ox. As
08. Uma partícula percorre o eixo Ox indicado, forças referidas têm a mesma direção do eixo.
deslocando-se da posição x1 = 2m para a posição
x2 = 8m:

Sobre ela, agem duas forças constantes, F1 e


F2, de intensidades respectivamente iguais a
Calcule:
80N e 10N. Calcule os trabalhos de F1 e F2 no a) o trabalho da força F1, enquanto o corpo é
deslocamento de x1 a x2. arrastado nos primeiros 10m;
b) o trabalho da força F2, enquanto o corpo é
arrastado nos primeiros 10m;
09. O bloco da figura acha-se inicialmente em repouso, c) o trabalho da força resultante, para arrastar o
corpo nos primeiros 15m.
livre da ação de forças externas. Em dado instante,
aplica-se sobre ele o sistema de forças indicado, 12. Um projétil de massa m é lançado obliquamente no
constituído por F1 , F2 , F3 e F4 , de modo que vácuo, descrevendo a traietória indicada abaixo:
F1 e F3 sejam perpendiculares a F4:

A altura máxima atingida é h e o módulo da


aceleração da gravidade vale g. Os trabalhos do
peso do projétil nos deslocamentos de A até B, de
B até C e de A até C valem, respectivamente:
a) 0, 0 e 0.
Sendo τ1, τ2, τ3 e τ4, respectivamente, os trabalhos b) m g h, m g h e 2m g h.
de F1 , F2, F3 e F4 para um deslocamento de c) –mg h, mgh e 0.
d) m g h, –m g h e 0.
5,0m, calcule τ1, τ2, τ3 e τ4.
OS.:0424/13-Juliana
34
Prof. Rodrigo Lins

13. O trabalho total realizado sobre uma partícula 18. Um homem puxa a extremidade livre de uma
de massa 8,0kg foi de 256 J. Sabendo que a mola de constante elástica igual a 1,0 . 103 N/m,
velocidade inicial da partícula era de 6,0m/s, alongando-a 20cm. O trabalho da força elástica
calcule a velocidade final. da mola sobre a mão do homem vale:
a) 40J.
b) 20J.
14. Uma partícula de massa 900g, inicialmente c) –40J.
em repouso na posição x0 = 0 de um eixo Ox, d) –20J.
submete-se à ação de uma força resultante e) –2,0 . 105J.
paralela ao eixo. O gráfico abaixo mostra a
variação da intensidade da força em função da
19. Considere um bloco de massa M = 10kg que se
abscissa da partícula:
move sobre uma superfície horizontal com uma
velocidade inicial de 10m/s. No local, o efeito do

AFA – EFOMM – EEAR| FÍSICA


ar é desprezível e adota-se IgI = 10m/s2.
a) Qual o trabalho realizado pela força de atrito
para levar o corpo ao repouso?
b) Supondo que o coeficiente de atrito cinético
seja µ = 0,10, qual o intervalo de tempo
necessário para que a velocidade do bloco
seja reduzida à metade do seu valor inicial?
Determine:
a) o trabalho da força de x0 = 0 a x1 = 6m; 20. Um vagão, deslocando-se lentamente com
b) a velocidade escalar da partícula na posição velocidade v num pequeno trecho plano e
x2 = 8m. horizontal de uma estrada de ferro, choca-se
com um monte de terra e para abruptamente.
Em virtude do choque, uma caixa de madeira, de
15. Um pequeno objeto de massa 2,0kg, abandonado massa 100kg, inicialmente em repouso sobre o
de um ponto situado a 15m de altura em relação piso do vagão, escorrega e percorre uma distância
ao solo, cai verticalmente sob a ação da força de 2,0m antes de parar, como mostra a figura.
peso e da força de resistência do ar. Sabendo
que sua velocidade ao atingir o solo vale 15m/s,
calcule o trabalho da força de resistência do ar.
Dado: g = 10m/s2

16. Um corpo de massa 6,0kg é abandonado


Considerando g = 10m/s 2 e sabendo que o
de uma altura de 5,0m num local em que coeficiente de atrito dinâmico entre a caixa e o
g = 10m/s2. Sabendo que o corpo chega ao solo piso do vagão é igual a 0,40, calcule:
com velocidade de intensidade 9,0m/s, calcule a
a) a velocidade v do vagão antes de se chocar
quantidade de calor gerada pelo atrito com o ar. com o monte de terra;
b) a energia cinética da caixa antes de o vagão
se chocar com o monte de terra e o trabalho
17. Um helicóptero suspenso no ar, em repouso realizado pela força de atrito que atuou na
em relação ao solo, ergue por meio de um cabo caixa enquanto ela escorregava.
de aço, mantido vertical, uma caixa de massa
igual a 200kg que se desloca com velocidade
21. Um projétil de 10g de massa atinge horizontalmente
constante ao longo de um percurso de 10m. No
uma parede de alvenaria com velocidade de
local, g = 10m/s2. Sabendo que no deslocamento 120m/s, nela penetrando 20cm até parar.
citado as forças de resistência do ar realizam sobre Determine, em newtons, a intensidade média da
a caixa um trabalho de –1 400J, calcule o trabalho força resistente que a parede opõe à penetração
da força aplicada pelo cabo de aço sobre a caixa. do projétil.
35 OS.:0424/13-Juliana
Prof. Rodrigo Lins

22. Um garoto de massa 40kg partiu do repouso no 25. Na situação esquematizada na figura, a mola tem
ponto A do tobogã da figura abaixo, atingindo o massa desprezível, constante elástica igual a
ponto B com velocidade de 10m/s: 1,0.102 N/m e está inicialmente travada na posição
indicada, contraída de 50cm. O bloco, cuja massa
é igual a 1,0kg, está em repouso no ponto A,
simplesmente encostado na mola. O trecho AB do
plano horizontal é perfeitamente polido e o trecho
BC é áspero.

Admitindo IgI = 10m/s2 e desprezando o efeito


do ar, calcule o trabalho das forças de atrito que Em determinado instante, a mola é destravada
AFA – EFOMM – EEAR| FÍSICA

agiram no corpo do garoto de A até B. e o bloco é impulsionado, atingindo o ponto B


com velocidade de intensidade VB. No local,
a influência do ar é desprezível e adota-se
23. Uma esfera de massa 1,0kg, lançada com g = 10m/s2. Sabendo que o bloco para ao atingir
velocidade de 10m/s no ponto R da calha vertical, o ponto C, calcule:
encurvada conforme a figura, atingiu o ponto S, por a) o valor de VB;
onde passou com velocidade de 4,0m/s: b) o coeficiente de atrito cinético entre o bloco e
o plano de apoio no trecho BC.

26. Na situação representada nas figuras 1 e 2, a


mola tem massa desprezível e está fixa no solo
com o seu eixo na vertical. Um corpo de pequenas
dimensões e massa igual a 2,0kg é abandonado
da posição A e, depois de colidir com o aparador
Sabendo que no local do experimento da mola na posição B, aderindo a ele, desce e
IgI = 10m/s2, calcule o trabalho das forças de para instantaneamente na posição C.
atrito que agiram na esfera no seu deslocamento
de R até S.

24. Um bloco de massa 2,0kg é lançado do topo de


um plano inclinado, com velocidade escalar de 5,0
m/s, conforme indica a figura. Durante a descida,
atua sobre o bloco uma força de atrito constante
de intensidade 7,5N, que faz o bloco parar após
deslocar-se 10m. Calcule a altura H, desprezando
o efeito do ar e adotando g = 10m/s2.

Adotando g = 10m/s2 e desprezando o efeito do ar


e a energia mecânica dissipada no ato da colisão,
calcule:
a) o trabalho do peso do corpo no percurso AC;
b) o trabalho da força aplicada pela mola sobre
o corpo no percurso BC;
c) a constante elástica da mola.
OS.:0424/13-Juliana
36
Prof. Rodrigo Lins

27. Na figura, um operário ergue um balde cheio de 31. As águas do rio São Francisco são represadas
concreto, de 20kg de massa, com velocidade em muitas barragens, para o aproveitamento
constante. A corda e a polia são ideais e, no local, do potencial hidrográfico e transformação de
g = 10m/s 2. Considerando um deslocamento energia potencial gravitacional em outras formas
vertical de 4,0m, que ocorre em 25s, determine: de energia. Uma dessas represas é Xingó,
responsável por grande parte da energia elétrica
que consumimos. A figura abaixo representa a
barragem e uma tubulação, que chamamos de
tomada d'água, e o gerador elétrico. Admita que,
no nível superior do tubo, a água está em repouso,
caindo a seguir até um desnível de 118m, onde
encontra o gerador de energia elétrica. O volume
de água que escoa, por unidade de tempo, é de
5,0 . 10m3/s. Considere a densidade da água igual
a 1,0 . 103kg/m3, adote g = 10m/s2 e admita que

AFA – EFOMM – EEAR| FÍSICA


não haja dissipação de energia mecânica.

a) o trabalho realizado pela força do operário;


b) a potência média útil na operação.

28. Dispõe-se de um motor com potência útil de 200W


para erguer um fardo de massa de 20kg à altura
de 100m em um local onde g = 10m/s2. Supondo
que o fardo parte do repouso e volta ao repouso,
calcule: Calcule, em MW, a potência hídrica na entrada do
gerador.
a) o trabalho desenvolvido pela força aplicada
pelo motor;
b) o intervalo de tempo gasto nessa operação.
32. No esquema abaixo, F é a força motriz que age
no carro e v , sua velocidade vetorial instantânea:
29. Um homem usa uma bomba manual para
extrair água de um poço subterrâneo a 60m de
profundidade. Calcule o volume de água, em litros,
que ele conseguirá bombear caso trabalhe com
potência constante de 50W durante 10 minutos.
Despreze todas as perdas e adote g = 10m/s2 e a
densidade da água igual a 1,0kg/L. Sendo |F| = 1,0 . 103N e |v| = 5,0m/s, calcule, em
kW, a potência de F no instante considerado.

30. O gráfico abaixo mostra a variação da intensidade


de uma das forças que agem em uma partícula em 33. Um gerador elétrico suposto ideal é acionado
função de sua posição sobre uma reta orientada. pela queda de um bloco de massa M que desce
A força é paralela à reta. Sabendo que a partícula sob a ação da gravidade com velocidade escalar
tem movimento uniforme com velocidade de constante de 5,0m/s. Sabendo que a potência
4,0m/s, calcule, para os 20m de deslocamento fornecida pelo gerador é usada para acender uma
descritos no gráfico a seguir: lâmpada de 100W, calcule o valor de M.

a) o trabalho da força;
b) sua potência média. Despreze os atritos e adote Ig| = 10 m/s2.
37 OS.:0424/13-Juliana
Prof. Rodrigo Lins

34. O diagrama abaixo representa a potência


instantânea fornecida por uma máquina, desde
t0 = 0s até t1 = 30s:

a) 2m . s–1
b) 5m . s–1
c) 6m . s–1
d) 10m . s–1
Com base no diagrama, determine: e) 15m . s–1
a) o trabalho realizado pela máquina, de t0 = 0s
AFA – EFOMM – EEAR| FÍSICA

até t1 = 30s; 38. O motor da figura leva o bloco de 10kg da posição


b) a potência média fornecida pela máquina no A para a posição B, com velocidade constante, em
intervalo referido no item anterior. 10s. O coeficiente de atrito cinético entre o bloco
e o plano inclinado é 0,50. Adote g = 10m/s2.
35. Um carro de massa 1,0 tonelada sobe 20m ao
longo de uma rampa inclinada de 20º com a
horizontal, mantendo velocidade constante de
10m/s. Adotando g = 10m/s2, sen 20° = 0,34
e cos 20º = 0,94 e desprezando o efeito do ar,
calcule, nesse deslocamento:
a) o trabalho realizado pelo peso do carro;
b) a potência útil do motor.
Qual a potência útil do motor nesse deslocamento?

36. Na figura, AB é um plano inclinado sem atrito e BC


é um plano horizontal áspero. Um pequeno bloco 39. Na situação da figura abaixo, o motor elétrico
parte do repouso no ponto A e para no ponto C: faz com que o bloco de massa 30kg suba com
velocidade constante de 1,0m/s. O cabo que
sustenta o bloco é ideal, a resistência do ar é
desprezível e adota-se IgI = 10m/s2. Considerando
que nessa operação o motor apresenta rendimento
de 60%, calcule a potência por ele dissipada.

Sabendo que o coeficiente de atrito cinético entre


o bloco e o plano BC vale 0,40 e que a influência
do ar é desprezível, calcule a distância percorrida
pelo bloco nesse plano.

37. Um bloco de peso igual a 10N parte do repouso


e sobe a rampa indicada na figura 1 mediante a
aplicação da força F, de direção constante e cuja
intensidade varia com a abscissa x, de acordo 40. Uma partícula A tem massa M e desloca-se
com o gráfico da figura 2. O trabalho de O até verticalmente para cima com velocidade de
A realizado pelo atrito existente entre o bloco e módulo v. Uma outra partícula B tem massa 2M e
a rampa é igual a 10J, em valor absoluto. Adote desloca-se horizontalmente para a esquerda com
g = 10 m.s–2. Nessas condições, a velocidade do velocidade de módulo v/2 . Qual a relação entre
bloco, ao atingir o ponto culminante A, é igual a: as energias cinéticas das partículas A e B?
OS.:0424/13-Juliana
38
Prof. Rodrigo Lins

41. Uma pequena pedra de massa 2,0kg acha-se 46. Um corpo movimenta-se sob a ação exclusiva
no fundo de um poço de 10m de profundidade. de forças conservativas. Em duas posições, A e
Sabendo que, no local, a aceleração da gravidade B, de sua trajetória, foram determinados alguns
tem módulo 10m/s2, indique a alternativa que traz valores de energia. Esses valores se encontram
o valor correto da energia potencial de gravidade na tabela abaixo:
da pedra em relação à borda do poço.
Energia Energia Energia
a) –2,0 . 102J.
cinética potencial mecânica
b) 2,0 . 102J. (joules) (joules (joules)
c) –20J.
Posição A 800 1000
d) 20J.
Posição B 600
e) Nenhuma das anteriores.
Os valores da energia cinética em A e das energias
potencial e mecânica em B são, respectivamente:
42. Tracionada com 800N, certa mola helicoidal
sofre distensão elástica de 10cm. Qual a energia a) 0 J, 800 J e 1 000 J.

AFA – EFOMM – EEAR| FÍSICA


potencial armazenada na mola quando deformada b) 200 J, 400 J e 1 000 J.
de 4,0cm? c) 100 J, 200 J e 800 J.
d) 200 J, 1 000 J e 400 J.
e) Não há dados suficientes para os cálculos.
43. Um corpo de massa m e velocidade v0 possui
energia cinética E0. Se o módulo da velocidade
aumentar em 20%, a nova energia cinética do 47. Um garoto de massa m = 30kg parte do repouso
corpo será: do ponto A do escorregador perfilado na figura
a) 1,56 E0. e desce, sem sofrer a ação de atritos ou da
b) 1,44 E0. resistência do ar, em direção ao ponto C:
c) 1,40 E0.
d) 1,20 E0.
e) 1,10 E0.

44. Uma bola de borracha de massa 1,0kg é


abandonada da altura de 10m. A energia perdida
por essa bola ao se chocar com o solo é 28J.
Supondo g = 10 m/s2, a altura máxima atingida
pela bola após o choque com o solo será de:
Sabendo que H = 20m e que IgI = 10m/s2, calcule:
a) 7,2m. d) 4,2m.
a) a energia cinética do garoto ao passar pelo
b) 6,8m. e) 2,8m.
ponto B;
c) 5,6m.
b) a intensidade de sua velocidade ao atingir o
ponto C.
45. O gráfico representa a energia cinética de uma
partícula de massa 10g, sujeita somente a forças
conservativas, em função da abscissa x. A energia 48. Numa montanha-russa, um carrinho com 300kg de
mecânica do sistema é de 400J. massa é abandonado do repouso de um ponto A,
que está a 5,0m de altura. Supondo que os atritos
sejam desprezíveis e que g = 10m/s2, calcule:

a) Qual a energia potencial para x = 1,0m e para


x = 4,0m? a) o valor da velocidade do carrinho no ponto B;
b) Calcule a velocidade da partícula para b) a energia cinética do carrinho no ponto C, que
x = 8,0m. está a 4,0m de altura.
39 OS.:0424/13-Juliana
Prof. Rodrigo Lins

49. A pista vertical representada é um quadrante Considerando-se a mola ideal e desprezando-se


de circunferência de 1,0m de raio. Adotando a resistência do ar, quanto subiria o disco se o
g = 10m/s 2 e considerando desprezíveis as garoto contraísse a mola de 10mm?
forças dissipativas, um corpo lançado em A com a) 400mm d) 80mm
velocidade de 6,0m/s desliza pela pista, chegando
b) 200mm e) 90mm
ao ponto B com velocidade:
c) 100mm

52. Um garoto de massa 40kg parte do repouso de


a) 6,0m/s
uma altura de 10m, desliza ao longo de um tobogã
b) 4,0m/s
e atinge a parte mais baixa com velocidade de
c) 3,0m/s
5,0m/s:
d) 2,0m/s
e) nula
AFA – EFOMM – EEAR| FÍSICA

50. No arranjo experimental da figura, desprezam-se


o atrito e o efeito do ar:

Admitindo a aceleração da gravidade igual a


10m/s2, calcule a energia mecânica degradada
O bloco (massa de 4,0kg), inicialmente em pelas forças dissipativas, durante a descida do
repouso, comprime a mola ideal (constante garoto.
elástica de 3,6 . 10 3N/m) de 20cm, estando
apenas encostado nela. Largando-se a mola,
esta distende-se impulsionando o bloco, 53. O carrinho de montanha-russa da figura seguinte
que atinge a altura máxima h. Adotando pesa 6,50 . 103N e está em repouso no ponto A,
IgI = 10m/s2, determine: numa posição de equilíbrio instável. Em dado
instante, começa a descer o trilho, indo atingir o
a) o módulo da velocidade do bloco imediatamente
ponto B com velocidade nula:
após desligar-se da mola.
b) o valor da altura h.

51. Um garoto brinca com uma mola helicoidal. Ele


coloca a mola em pé em uma mesa e apoia sobre
ela um pequeno disco de plástico. Segurando a
borda do disco, ele comprime a mola, contraindo-a
de 5mm. Após o garoto soltar os dedos, a mola
projeta o disco 100mm para cima (contados do
ponto de lançamento, veja a figura).

Sabendo que a energia térmica gerada pelo atrito


de A até B equivale a 4,55 . 104J, determine o valor
da altura h.

54. Uma partícula de massa 1,0kg é lançada


verticalmente para cima com velocidade de
módulo 20m/s num local em que a resistência do
ar é desprezível e IgI = 10m/s2. Adotando o nível
horizontal do ponto de lançamento como plano de
referência, calcule:
a) a energia mecânica da partícula;
b) a altura do ponto em que a energia cinética é
o triplo da potencial de gravidade.

OS.:0424/13-Juliana
40
Prof. Rodrigo Lins

55. Um jogador de voleibol, ao dar um saque, 58. Um bloco de massa m = 0,60kg, sobre um trilho
comunica à bola uma velocidade inicial de 10m/s. de atrito desprezível, comprime uma mola de
A bola, cuja massa é de 400g, passa a se mover
constante elástica k = 2,0 . 103N/m, conforme a
sob a ação exclusiva do campo gravitacional
(IgI = 10 m/s2), descrevendo a trajetória indicada figura abaixo.
na figura:

Considere que a energia potencial gravitacional


Calcule: seja zero na linha tracejada. 0 bloco, ao ser

AFA – EFOMM – EEAR| FÍSICA


a) a energia mecânica da bola no ponto A em
liberado, passa pelo ponto P (h = 0,60m),
relação ao plano de referência indicado.
b) o módulo da velocidade da bola ao passar pelo onde 75% de sua energia mecânica é cinética.
ponto B (mais alto da trajetória). Adote g = 10,0m/s2 e despreze o efeito do ar. A
compressão x da mola foi de:
56. Um pequeno bloco B, lançado do ponto P com a) 9,0cm
velocidade de intensidade v0, desliza sem atrito e
b) 12,0cm
sem sofrer influência do ar sobre a superfície PQ,
contida em um plano vertical. c) 15,0cm
d) 18,0cm
e) 21,0cm

59. Na figura abaixo, uma esfera de massa


m = 5,0kg é abandonada do ponto R no instante
t1, caindo livremente e colidindo com o aparador,
que está ligado a uma mola de constante elástica
Sabendo que B inverte o sentido do movimento no igual a 2,0 . 103N/m. As massas da mola e do
ponto Q e que IgI = 10m/s2, calcule o valor de v0. aparador são desprezíveis, como também o são
todas as dissipações de energia mecânica.

57. Uma bolinha é abandonada do ponto A do trilho


liso AB e atinge o solo no ponto C. Supondo que a
velocidade da bolinha no ponto B seja horizontal,
a altura h vale:

Considerando g = 10m/s 2 e supondo que no


instante t2 a mola está sob compressão máxima,
calcule:

a) 1,25m d) 2,25m a) a compressão da mola quando a esfera atinge


b) 1,75m e) 2,50m sua máxima velocidade.
c) 2,00m b) a compressão da mola no instante t2.
41 OS.:0424/13-Juliana
Prof. Rodrigo Lins

60. Um corpo de massa 1,0kg cai livremente, a 63. Uma pequena conta de vidro de massa igual a 10g
partir do repouso, da altura y = 6,0m sobre uma desliza sem atrito ao longo de um arame circular
mola de massa desprezível e eixo vertical, de de raio R = 1,0m, como indicado na figura.
constante elástica igual a 1,0 . 102 N/m. Adotando
g = 10 m/s2 e desprezando todas as dissipações de
energia mecânica, calcule a máxima deformação
x da mola.

Se a conta partiu do repouso na posição A,


determine o valor de sua energia cinética ao
passar pelo ponto B. O arame está posicionado
AFA – EFOMM – EEAR| FÍSICA

verticalmente em um local em que IgI = 10m/s2.

64. A mola da figura abaixo possui uma constante


elástica K = 280 N/m e está inicialmente comprimida
61. O pêndulo da figura oscila para ambos os lados, de 10cm:
formando um ângulo máximo de 60° com a vertical:

O comprimento do fio é de 90cm e, no local, o Uma bola com massa de 20g encontra-se


módulo da aceleração da gravidade vale 10m/s2. encostada na mola no instante em que esta é
Supondo condições ideais, determine: abandonada. Considerando g = 10m/s2 e que
a) o módulo da velocidade da esfera no ponto todas as superfícies são perfeitamente lisas,
mais baixo de sua trajetória;
determine:
b) a intensidade da força que traciona o fio
quando este se encontra na vertical (adotar, a) o valor da velocidade da bola no ponto D.
para a massa da esfera, o valor 50g). b) o valor da força que o trilho exerce na bola no
ponto D.
62. A figura mostra um trecho de uma montanha-russa c) o valor da aceleração tangencial da bola
de formato circular de raio R. Um carro de massa quando ela passa pelo ponto C.
M = 200kg parte do repouso de uma altura R

65. No esquema da figura, o bloco tem massa 3,0kg


e encontra-se inicialmente em repouso num ponto
da rampa, situado à altura de 1,0m:

Considere o instante em que o carro passa pelo


ponto mais baixo da trajetória (ponto B). Despreze Uma vez abandonado, o bloco desce atingindo a
as forças de atrito e use g = 10m/s2. mola de constante elástica igual a 1,0 . 103N/m,
a) Faça uma figura representando as forças que que sofre uma compressão máxima de 20cm.
atuam sobre o carro nesse instante. Adotando IgI = 10m/s2, calcule a energia mecânica
b) Calcule a intensidade da força que a pista faz dissipada no processo.
sobre ele nesse instante.
OS.:0424/13-Juliana
42
Prof. Rodrigo Lins

66. Uma pequena esfera penetra com velocidade V em Gabarito dos Exercícios
um tubo oco, recurvado e colocado em um plano
vertical, como mostra a figura, num local onde a
aceleração da gravidade tem módulo igual a g. 01 02 03 04 05 06 07 08
Supondo que a esfera percorra a região interior
* * c * * * * *
do tubo sem atrito e acabe saindo horizontalmente
pela extremidade, pergunta-se: que distância x, 09 10 11 12 13 14 15 16
horizontal, ela percorrerá até tocar o solo? * * * c * * * *
17 18 19 20 21 22 23 24
* d * * * * * *
25 26 27 28 29 30 31 32
* * * * * * * *
33 34 35 36 37 38 39 40
* * * * d * * *

AFA – EFOMM – EEAR| FÍSICA


41 42 43 44 45 46 47 48
a * b a * b * *
49 50 51 52 53 54 55 56
b * a * * * * *
57 58 59 60 61 62 63 64
a b * * * * * *
65 66 67
* * *
67. Na figura, tem-se um cilindro de massa 5,0kg,
dotado de um furo, tal que, acoplado à barra vertical
indicada, pode deslizar sem atrito ao longo dela. 01. nulo
Ligada ao cilindro, existe uma mola de constante 02. 4500J
elástica igual a 5,0 . 102N/m e comprimento natural
de 8,0cm, cuja outra extremidade está fixada no 04. a) 50N b) zero
ponto O. Inicialmente, o sistema encontra-se em 05. a) 120J b) -80J c) 40J
repouso (posição A) quando o cilindro é largado,
06. 3200J
descendo pela barra e alongando a mola. Calcule
o módulo da velocidade do cilindro depois de ter 07. a) µd = 0,15; µc = 0,10 b) 100J c) nulo
descido 16cm (posição B). Adote nos cálculos
08. 240J e - 60J
g = 10m/s2.
09. zero, 300J, zero e -250J
10. 600J
11. a) 400J b) -100J c) 600J
13. 10m/s
14. a) 45J b) 10m/s
15. -75J
16. 57J
17. 21400J
19. a) -500J b) 1m/s2 e 5s
20. a) 4m/s b) 800J e -800J
21. 360N
22. -2000J
23. -22J
24. 2,5m
25. a) 5m/s b) 0,25
43 OS.:0424/13-Juliana
Prof. Rodrigo Lins

26. a) 5J b) -5J c) 4000N/m Quantidade de movimento e sua


27. a) 800J b) 32W conservação
28. a) 20000J b) 100s
29. 45 litros 1. Impulso de uma força constante
30. a) 550J b) 110W Ao se disparar um tiro, o projétil de um fuzil é
31. 590 MW impulsionado pelos gases provenientes da detonação
do explosivo. Esses gases agem muito rapidamente
32. 5KW sobre o projétil, porém, de forma muito intensa,
33. 2kg determinando um impulso considerável.
Em nosso curso, iremos nos restringir à definição
34. a) 45000J b) 1500W do impulso de uma força constante (intensidade,
direção e sentido invariáveis), uma vez que a
35. a) -68000J b) 34KW
definição geral dessa grandeza requer elementos de
36. 5m Matemática normalmente não estudados no Ensino
AFA – EFOMM – EEAR| FÍSICA

Médio.
38. 20W
39. 200W
40. 2
O impulso de F intervalo de tempo (∆t = t2 – t1) é
42. 6,4J
a grandeza vetorial definida por:
45. a) 400J e zero b) 200m/s
47. a) 4KJ b) 20m/s  
I= F ∆t
48. a) 10m/s b) 3KJ
Unidades:
50. a) 6m/s b) 1,8m
unid (I) =unid (F) . unid (∆t)
52. 3500J
Impulso= 1 N . s
53. 13m
54. a) 200J b) 5m 2. Cálculo gráfico do valor algébrico do impulso

55. a) 20J b) 6m/s Dado um diagrama do valor algébrico da força


atuante em uma partícula em função do tempo, a
56. 8m/s "área" compreendida entre o gráfico e o eixo dos
59. a) 2,5cm b) 50cm tempos expressa o valor algébrico do impulso da
força. No entanto, a força considerada deve ter
60. 1,2m direção constante.
61. a) 3m/s b) 1N
62. b) 4KN
63. 0,05J A=I

64. a) 10m/s b) 1,8N c) 10m/s2


65. 10J 3. Quantidade de movimento(Q)
3R 2 Em diversos fenômenos físicos, é necessário
66. (V + q R) agrupar os conceitos de massa e de velocidade
g
vetorial.
Isso ocorre nas colisões mecânicas e nas
67. 1,4m/s explosões, por exemplo. Nesses casos, torna-se
conveniente a definição de quantidade de movimento
(ou momento linear), que é uma das grandezas
fundamentais da Física.
Por definição, a quantidade de movimento da
partícula nesse instante é a grandeza vetorial Q,
expressa por:
 
Q =m v

OS.:0424/13-Juliana
44
Prof. Rodrigo Lins

Nota: não seriam resultantes, uma vez que cada patinador,


pela ação da força recebida, tem seu corpo acelerado
• A quantidade de movimento é uma grandeza
a partir do repouso? E a resposta é simples: sim,
instantânea, já que sua definição envolve o essas forças (ação e reação) são as resultantes que
conceito de velocidade vetorial instantânea. aceleram cada corpo, porém são forças internas ao
• Sendo m um escalar positivo, Q tem sempre a sistema, não devendo ser consideradas no estudo do
mesma direção e o mesmo sentido de V. sistema como um todo.

Para m constante, Q tem módulo diretamente


proporcional ao módulo de V. O gráfico a seguir 6. Quantidade de movimento e energia mecânica
representa tal proporcionalidade: nas colisões
Em qualquer tipo de colisão mecânica, a
 quantidade de movimento total do sistema mantém-se
|Q| constante. A quantidade de movimento imediatamente
tg=
α =  m após a interação é igual à quantidade de movimento
|v|
imediatamente antes:

AFA – EFOMM – EEAR| FÍSICA


 
Qfinal = Qinicial
• É interessante ressaltar que, quanto maior for a
intensidade da quantidade de movimento de um 7. O coeficiente de restituição ou de elasticidade
corpo, maior será seu "poder de impacto". (e)
O coeficiente de restituição ou de elasticidade (e)
4. O teorema do impulso para a referida colisão é definido pelo quociente:
| v raf |
O impulso da resultante (impulso total) das e=
forças sobre uma partícula é igual à variação de sua | v rap |
quantidade de movimento:
    
Itotal =
∆Q ⇒ Itotal =
Qfinal − Qinicial Notas:
• O coeficiente de restituição (e) não depende da
Nota: massa, mas depende dos materiais dos corpos
• Devemos observar apenas que a força, cujo que participam da colisão.
impulso é igual à variação da quantidade de • O coeficiente de restituição (e) é adimensional por
movimento, deve ser a resultante. ser calculado pelo quociente de duas grandezas
• O impulso da força resultante é equivalente à soma medidas nas mesmas unidades.
• Pode-se demonstrar que:

5. Sistema mecânico isolado 0≤e≤1


Um sistema mecânico é denominado isolado 8. Classificação das colisões quanto ao valor do
de forças externas quando a resultante dos forças coeficiente de restituição
externas atuantes sobre ele for nula.
Admita que dois patinadores, inicialmente
em repouso sobre uma plataforma de gelo plana e 8.1 Colisões elásticas (ou perfeitamente elásticas)
horizontal, empurrem-se mutuamente, conforme
sugere a figura. São aquelas em que o coeficiente de restituição
máximo, isto é:

e=1 ⇒ |vr | = |vr |


af ap

Numa colisão elástica, as partículas


Desprezando os atritos e a influência do ar, os
aproximam-se (antes da colisão) e afastam-se (depois
dois patinadores constituem um sistema mecânico
da colisão) com a mesma velocidade escalar relativa
isolado, pois a resultante das forças externas atuantes
em módulo.
no conjunto é nula. De fato, as únicas forças externas
que agem em cada patinador são a força da gravidade
(peso) e a força de sustentação da plataforma de gelo Colisão elástica ⇒ Sistema conservativo
(normal), que se equilibram. EC = EC
Entretanto, uma pergunta surge naturalmente: final inicial
as forças trocadas entre eles no ato do empurrão

45 OS.:0424/13-Juliana
Prof. Rodrigo Lins

8.2 Colisões inelásticas. 02. Uma partícula de massa m = 10g se move no plano
x, y com uma velocidade tal que sua componente,
São aquelas em que o coeficiente de restituição
ao longo do eixo x, é de 4,0m/s e, ao longo do eixo
é nulo:
y, é de 2,0m/s. Nessas condições, pode-se afirmar
que sua energia cinética vale:
e=0 ⇒ |vr | = 0 a) 0,10J
af
b) 0,18J
Nas colisões totalmente inelásticas, como a c) 100J
velocidade escalar relativa de afastamento tem d) 180J
módulo nulo, concluímos que, após a interação, os e) 190J
corpos envolvidos não se separam.

03. Uma ema pesa aproximadamente 360N e


Colisão totalmente inelástica ⇒ Sistema dissipativo consegue desenvolver uma velocidade de 60km/h,
EC < EC o que lhe confere uma quantidade de movimento
AFA – EFOMM – EEAR| FÍSICA

final inicial
linear, em kg.m/s, de
Dado: aceleração da gravidade = 10m/s2
10.3 Colisões parcialmente inelásticas. a) 36
b) 360
São aquelas em que o coeficiente de restituição
situa-se entre zero e um: c) 600
d) 2 160
e) 3 600
0<e<1 ⇒ 0 < |vr | < |vr |
af ap

04. Um vendedor, antes de fazer um embrulho, enrola


Nas colisões parcialmente elásticas, os corpos cada uma das extremidades de um pedaço de
envolvidos separam-se após a interação, existindo, barbante em cada uma das mãos e, em seguida,
assim, a fase de restituição. Os corpos afastam-se, as afasta tentando romper o barbante. Para o
entretanto, com velocidade escalar relativa de módulo mesmo tipo de barbante, é mais fácil conseguir
o rompimento com um movimento brusco do que
menor que o da aproximação.
com um movimento progressivo. Isto se deve a
variação, em um intervalo de tempo muito curto,
Colisão parcialmente elástica ⇒ Sistema dissipativo da seguinte grandeza física associada às mãos:
EC < EC a) energia
final inicial
b) velocidade
c) aceleração
d) momento linear

05. Num certo instante, um corpo em movimento tem


energia cinética de 100 joules, enquanto o módulo
Exercícios de Fixação de sua quantidade de movimento é 40kg.m/s.

01. Uma esfera se move sobre uma superfície


horizontal sem atrito. Num dado instante, sua
energia cinética vale 20J e sua quantidade de
movimento tem módulo 20 N.s. Nestas condições,
é correto afirmar que sua: A massa do corpo, em kg, é
a) velocidade vale 1,0m/s. a) 5,0
b) velocidade vale 5,0m/s. b) 8,0
c) velocidade vale 10m/s. c) 10
d) 16
d) massa é de 1,0kg.
e) 20
e) massa é de 10kg.
OS.:0424/13-Juliana
46
Prof. Rodrigo Lins

06. Dois patinadores, um de massa 100kg e outro 10. Dois carrinhos para experiências no laboratório de
de massa 80kg, estão de mãos dadas em Física são idênticos e têm um engate que obriga
repouso sobre uma pista de gelo, onde o atrito um carrinho a ficar preso ao outro quando se
é desprezível. Eles empurram-se mutuamente e chocam. Pretende-se colocar um dos carrinhos
deslizam na mesma direção, porém em sentidos em movimento retilíneo, com velocidade escalar
opostos. O patinador de 100kg adquire uma V, para chocar-se com o outro e imprimir-lhe uma
velocidade de 4m/s. A velocidade relativa de um velocidade de 1,0m/s. Após o choque, ambos
dos patinadores em relação ao outro é, em módulo, seguem unidos. O valor da velocidade escalar V
igual a: do primeiro carrinho, em m/s, deve ser, no mínimo,
igual a:
a) 5m/s
a) 0,50 d) 1,5
b) 4m/s
b) 0,75 e) 2,0
c) 1m/s
c) 1,0
d) 9m/s
e) 20m/s

AFA – EFOMM – EEAR| FÍSICA


11. Um ciclista, juntamente com sua bicicleta,
tem massa de 80kg. Partindo do repouso
07. Um observador, situado em um sistema de
de um ponto do velódromo, ele acelera com
referência inercial, constata que um corpo de
aceleração escalar constante de 1,0m/s2. Calcule
massa igual a 2kg, que se move com velocidade
o módulo da quantidade de movimento do sistema
constante de 15m/s no sentido positivo do eixo x,
ciclista-bicicleta decorridos 20s da partida.
recebe um impulso de 40N.s em sentido oposto
ao de sua velocidade. Para esse observador, com
que velocidade, especificada em módulo e sentido, 12. Uma partícula de massa 8,0kg desloca-se em
o corpo se move imediatamente após o impulso? trajetória retilínea, quando lhe é aplicada, no
a) –35m/s. sentido do movimento, uma força resultante de
b) 35m/s. intensidade 20N. Sabendo que no instante de
c) –10m/s. aplicação da força a velocidade da partícula valia
d) –5m/s 5,0m/s, determine:
e) 5m/s a) o módulo do impulso comunicado à partícula,
durante 10s de aplicação da força;
b) o módulo da velocidade da partícula ao fim do
08. Um jogador de tênis recebe uma bola com intervalo de tempo referido no item anterior.
velocidade de 20,0m/s e a rebate na mesma
direção e em sentido contrário com velocidade de
30,0m/s. Se a bola permanecer 0,100s em contato 13. Uma bola de bilhar de massa 0,15kg, inicialmente
com a raquete, o módulo da sua aceleração média em repouso, recebeu uma tacada numa direção
será de: paralela ao plano da mesa, o que lhe imprimiu
uma velocidade de módulo 4,0m/s. Sabendo que
a) 100m/s2
a interação do taco com a bola durou 1,0 . 10–2s,
b) 200m/s2
calcule:
c) 300 m/s2
d) 500m/s2 a) a intensidade média da força comunicada pelo
taco à bola.
e) 600m/s2
b) a distância percorrida pela bola, enquanto em
contato com o taco.
09. Um caminhão a 90km/h colide com a traseira
de um automóvel que viaja com movimento de
14. Um corpo de massa m = 10kg se movimenta sobre
mesmo sentido e velocidade 54km/h. A massa
uma superfície horizontal perfeitamente polida,
do caminhão é o triplo da massa do automóvel.
com velocidade escalar v o = 4,0m/s, quando
Imediatamente após a colisão, os dois veículos
uma força constante de intensidade igual a 10N
caminham juntos, com velocidade de:
passa a agir sobre ele na mesma direção do
a) 66km/h movimento, porém em sentido oposto. Sabendo
b) 68km/h que a influência do ar é desprezível e que quando a
c) 72km/h força deixa de atuar a velocidade escalar do corpo
d) 78km/h é v = –10 m/s, determine o intervalo de tempo de
e) 81km/h atuação da força.
47 OS.:0424/13-Juliana
Prof. Rodrigo Lins

15. Um corpo de massa 38kg percorre um eixo 18. Um menino faz girar uma pedra presa a uma haste
orientado com velocidade escalar igual a 15m/s. rígida e de massa desprezível de maneira que ela
No instante to = 0, aplica-se sobre ele uma força descreva um movimento circular uniforme num
resultante cujo valor algébrico varia em função do plano vertical, num local onde a aceleração da
tempo, conforme o gráfico abaixo: gravidade é constante. Sobre esse movimento,
considere as seguintes grandezas relacionadas
com a pedra:
I. Quantidade de movimento.
II. Energia potencial de gravidade.
III. Energia cinética.
IV. Peso
Admitindo que a força seja paralela ao eixo, calcule
a velocidade escalar do corpo no instante t = 14s.
Dentre essas grandezas, as que variam, enquanto
a pedra realiza seu movimento, são:
AFA – EFOMM – EEAR| FÍSICA

16. Um carrinho de massa 2,0kg encontra-se


inicialmente em repouso sobre um plano horizontal a) apenas I e IV. d) apenas Ill e IV.
sem atrito. A partir do instante t0 = 0, passa a b) apenas I e lI. e) apenas I e III.
agir sobre ele uma força F de direção constante, c) apenas II e III.
paralela ao plano, cujo valor algébrico é dado em
função do tempo, conforme o gráfico abaixo: 19. A um pequeno bloco que se encontra inicialmente
em repouso sobre uma mesa horizontal e lisa
aplica-se uma força constante, paralela à mesa,
que lhe comunica uma aceleração de 5,0m/s2.
Observa-se, então, que, 4,0s após a aplicação da
força, a quantidade de movimento do bloco vale
40kgm/s. Calcule, desprezando o efeito do ar, o
trabalho da força referida desde sua aplicação até
o instante t = 4,0s.

Desprezando a influência do ar, determine as


velocidades escalares do carrinho nos instantes
t1 = 2,0s, t2 = 4,0s e t3 = 6,0s.
Exercícios Propostos
01. Uma partícula de massa igual a 2,0kg,
17. Um garoto chuta uma bola e esta descreve uma
inicialmente em repouso sobre o solo, é puxada
trajetória parabólica, como representa a figura,
verticalmente para cima por uma força constante
sob a ação exclusiva do campo gravitacional,
F, de intensidade 30N, durante 3,0s. Adotando
considerado uniforme:
g = 10m/s2 e desprezando a resistência do ar,
calcule a intensidade da velocidade da partícula
no fim do citado intervalo de tempo.

02. Uma bola de massa igual a 40g, ao chegar ao local


em que se encontra um tenista, tem velocidade
Indique a alternativa cuja seta melhor representa horizontal de módulo 12m/s. A bola é golpeada
a variação da quantidade de movimento da bola pela raquete do atleta, com a qual interage durante
entre os pontos A e B: 2,0 . 10–2s, retornando horizontalmente em sentido
a) oposto ao do movimento inicial. Supondo que
b) a bola abandone a raquete com velocidade de
módulo 8,0m/s, calcule a intensidade média da
c) força que a raquete exerce sobre a bola.
+

–12m/s 8,0m/s
d) x1 x1
e) Faltam dados para uma conclusão possível. Antes Depois

OS.:0424/13-Juliana
48
Prof. Rodrigo Lins

03. Uma bola de tênis de massa m é lançada contra o Sabendo que em t0 = 0 (início do chute) a bola
solo, com o qual interage, refletindo-se em seguida estava em repouso, calcule:
sem perdas de energia cinética. O esquema
abaixo representa o evento: a) o módulo da quantidade de movimento da bola
no instante t1= 8,0 . 10–2s (fim do chute);
b) o trabalho realizado pela força que o pé do
jogador exerce na bola.

Sabendo que Ivi| = V e que a interação tem


duração ∆t, calcule a intensidade média da força 07. Considere o esquema abaixo, em que, inicialmente,
que o solo exerce na bola. tanto o homem quanto o carrinho estão em repouso
em relação ao solo. No local não há ventos e a
04. Considere um carro de massa igual a 8,0 . 102kg influência do ar é desprezível. O carrinho é livre

AFA – EFOMM – EEAR| FÍSICA


que entra em uma curva com velocidade v1 de
para se mover para a esquerda ou para a direita
intensidade 54km/h e sai dessa mesma curva com
velocidade v2 de intensidade 72km/h. Sabendo sobre trilhos horizontais, sem atrito.
que v2 é perpendicular a v1, calcule a intensidade
do impulso total (da força resultante) comunicado
ao carro.

05. Um carro de massa igual a 1,0 tonelada


percorre uma pista como a esquematizada na
figura, deslocando-se do ponto A ao ponto B Em determinado instante, o homem sai do ponto A
em movimento uniforme, com velocidade de e dirige-se para o ponto B, movendo-se na direção
intensidade igual a 90km/h. do eixo longitudinal do carrinho. Admitindo que, ao
chegar a B, o homem para em relação ao carrinho,
analise as seguintes proposições:

(01) A quantidade de movimento total do


sistema constituído pelo homem e pelo
carrinho é nula em qualquer instante.
Sabendo que o comprimento do trecho AB é igual
a 500m, calcule: (02) Enquanto o homem dirige-se do ponto

a) o intervalo de tempo gasto pelo carro no A para o ponto B, sua quantidade de


percurso de A até B; movimento é não-nula e oposta à do
b) a intensidade da força capaz de provocar a carrinho.
variação de quantidade de movimento sofrida
pelo carro de A até B. (04) Enquanto o homem dirige-se do ponto A
para o ponto B, sua velocidade é não-nula
06. Ao cobrar uma falta, um jogador de futebol chuta e oposta à do carrinho.
uma bola de massa igual a 4,5 . 102g. No lance, (08) Ao atingir o ponto B, o homem para em
seu pé comunica à bola uma força resultante de
relação ao carrinho e este, por sua vez,
direção constante, cuja intensidade varia com o
tempo, conforme o seguinte gráfico: para em relação ao solo.
(16) Após a chegada do homem a B, o sistema
prossegue em movimento retilíneo e
uniforme, por inércia.

Dê como resposta a soma dos números associados


às proposições corretas.
49 OS.:0424/13-Juliana
Prof. Rodrigo Lins

08. Uma bomba, inicialmente em repouso, explode, 12. Um astronauta de massa 70kg encontra-se em
fragmentando-se em três partes que adquirem repouso numa região do espaço em que as ações
quantidades de movimento coplanares de gravitacionais são desprezíveis. Ele está fora de
sua nave, a 120m dela, mas consegue mover-se
intensidades iguais. Qual das alternativas a seguir
com o auxílio de uma pistola que dispara projéteis
melhor representa a situação das partes da bomba de massa 100g, os quais são expelidos com
imediatamente após a explosão? velocidade de 5,6 . 102m/s. Dando um único tiro,
qual o menor intervalo de tempo que o astronauta
leva para atingir sua nave, suposta em repouso?

13. Num rinque de patinação, dois patinadores, João,


com massa de 84kg, e Maria, com massa 56kg,
estão abraçados e em repouso sobre a superfície
do gelo, ligados por um fio inextensível de 10,0m
AFA – EFOMM – EEAR| FÍSICA

de comprimento. Desprezando-se o atrito entre


os patinadores e a superfície do gelo, é correto
afirmar que, se eles se empurrarem, passando
a descrever movimentos retilíneos uniformes
em sentidos opostos, a distância, em metros,
09. Sobre um plano horizontal e perfeitamente liso, percorrida por Maria, antes de o fio se romper, é:
repousam, frente a frente, um homem e uma caixa a) 4,0 d) 8,0
de massas respectivamente iguais a 80kg e 40kg. b) 5,0 e) 10,0
Em dado instante, o homem empurra a caixa, que c) 6,0
se desloca com velocidade de módulo 10m/s.
Desprezando a influência do ar, calcule o módulo 14. Uma menina de 40kg é transportada na garupa de
da velocidade do homem após o empurrão. uma bicicleta de 10kg, a uma velocidade constante
de módulo 2,0m/s, por seu irmão de 50kg. Em dado
instante, a menina salta para trás com velocidade
de módulo 2,5m/s em relação ao solo. Após o salto,
10. Um casal participa de uma competição de
o irmão continua na bicicleta, afastando-se da
patinação sobre o gelo. Em dado instante, o rapaz, menina. Qual o módulo da velocidade da bicicleta,
de massa igual a 60kg, e a garota, de massa em relação ao solo, imediatamente após o salto?
igual a 40kg, estão parados e abraçados frente Admita que durante o salto o sistema formado
a frente. Subitamente, o rapaz dá um empurrão pelos irmãos e pela bicicleta seja isolado de forças
na garota, que sai patinando para trás com uma externas.
velocidade de módulo igual a 0,60m/s. Qual o a) 3,0m/s d) 4,5m/s
módulo da velocidade do rapaz ao recuar, como b) 3,5m/s e) 5,0m/s
consequência desse empurrão? Despreze o atrito c) 4,0m/s
com o chão e o efeito do ar.
15. Um canhão, juntamente com o carrinho que lhe
serve de suporte, tem massa M. Com o conjunto
em repouso, dispara-se obliquamente um projétil
de massa m, que, em relação ao solo, desliga-se
do canhão com uma velocidade de módulo v0,
inclinada de um ângulo com a horizontal. A figura
abaixo retrata o evento:
11. Um homem de massa 70kg, sentado em uma
cadeira de rodas inicialmente em repouso sobre
o solo plano e horizontal, lança horizontalmente
um pacote de massa 2,0kg com velocidade de
intensidade 10m/s. Sabendo que, imediatamente
após o lançamento, a velocidade do conjunto
homem-cadeira de rodas tem intensidade igual a Desprezando os atritos, determine o módulo da
0,25m/s, calcule a massa da cadeira de rodas. velocidade de recuo do conjunto canhão-carrinho.

OS.:0424/13-Juliana
50
Prof. Rodrigo Lins

16. Um garoto de massa 48kg está de pé sobre um Em determinado instante, queima-se o barbante
skate de massa 2,0kg, inicialmente em repouso e a mola se expande, impulsionando os blocos.
sobre o solo plano e horizontal. Em determinado Sabendo que o bloco B adquire velocidade de
instante, ele lança horizontalmente uma pedra intensidade 3,0m/s e que a influência do ar é
de massa 5,0kg, que adquire uma velocidade de desprezível, determine:
afastamento (relativa ao garoto) de módulo 11m/s. a) a intensidade da velocidade adquirida pelo
Sendo vG e vP, respectivamente, os módulos da bloco A.
velocidade do garoto e da pedra em relação ao b) a energia potencial elástica armazenada na
solo imediatamente após o lançamento, calcule mola antes da queima do barbante.
vG e VP.
20. A figura representa duas esferas, 1 e 2, de massas
m1 e m2, respectivamente, comprimindo uma mola
17. Imagine a seguinte situação: um dálmata corre e
e sendo mantidas por duas travas dentro de um
pula para dentro de um pequeno trenó, até então
tubo horizontal.
parado, caindo nos braços de sua dona. Em

AFA – EFOMM – EEAR| FÍSICA


consequência, o trenó começa a se movimentar.
Considere os seguintes dados:
I. A massa do cachorro é de 10kg.
II. A massa do conjunto trenó + moça é de 90kg.
Ill. A velocidade horizontal do cachorro Quando as travas são retiradas simultaneamente,
imediatamente antes de ser agarrado por as esferas 1 e 2 são ejetadas do tubo, com
sua dona é de 18km/h. velocidades de módulos v1 e v2, respectivamente,
e caem sob a ação da gravidade. A esfera 1 atinge
o solo num ponto situado à distância x1 = 0,50m, t1
a) Desprezando-se o atrito entre o trenó e o gelo,
bem como a influência do ar, determine a segundos depois de abandonar o tubo, e a esfera
velocidade horizontal do sistema trenó + moça 2, à distância x2 = 0,75m, t2 segundos depois de
+ cachorro imediatamente após o cachorro ter abandonar o tubo, conforme indicado na figura
caído nos braços de sua dona. acima.
b) Determine a variação da energia cinética do
sistema no processo.
21. Na situação esquematizada na figura, um garoto
de massa 40kg está posicionado na extremidade
18. Um canhão montado em um carro de combate em A de uma prancha de madeira, de massa 120kg,
repouso dispara um projétil de massa m = 2,50kg dotada de rodas, que tem sua extremidade B em
com velocidade horizontal v = 200m/s. O conjunto contato com um muro vertical. O comprimento AB
canhão-carro tem massa M = 5,00 . 102kg. Mesmo da prancha é igual a 6,0m.
com as rodas travadas, o carro recua, arrastando
os pneus no solo, percorrendo uma distância
L = 0,250m até parar. A aceleração local da
gravidade é g = 9,75m/s2. Calcule o coeficiente
de atrito cinético entre os pneus e o solo.

19. Dois blocos A e B, de massas respectivamente


Em determinado instante, o garoto começa a
iguais a 2,0kg e 4,0kg, encontram-se em caminhar de A para B com velocidade de módulo
repouso sobre um plano horizontal perfeitamente 1,2m/s em relação à prancha. Admitindo que o
polido. Entre os blocos, há uma mola de massa sistema garoto-prancha seja isolado de forças
desprezível, comprimida, que está impedida de externas e que o garoto pare de caminhar ao
expandir-se devido a um barbante que conecta atingir a extremidade B, calcule:
os blocos.
a) o módulo da velocidade da prancha em relação
ao solo enquanto o garoto caminha de A para
B;
b) a distância x entre a extremidade B da prancha
e o muro no instante em que o garoto atinge
a extremidade B.
51 OS.:0424/13-Juliana
Prof. Rodrigo Lins

22. A figura abaixo representa um homem de massa 24. Considere uma espaçonave em movimento
60kg, de pé sobre uma prancha de madeira, de retilíneo, com velocidade escalar de 2,0 . 103m/s
massa 120kg, em repouso na água de uma piscina. numa região de influências gravitacionais
Inicialmente, o homem ocupa o ponto A, oposto de desprezíveis. Em determinado instante, ocorre
B, onde a prancha está em contato com a escada. uma explosão e a espaçonave se fragmenta em
duas partes, A e B, de massas respectivamente
iguais a M e 2M. Se a parte A adquire velocidade
escalar de 8,0 . 103 m/s, qual a velocidade escalar
adquirida pela parte B?

25. Uma bomba, em queda vertical nas proximidades


Em determinado instante, o homem começa a da superfície terrestre, explode no instante em
andar, objetivando alcançar a escada. Não levando que a intensidade de sua velocidade é 20m/s. A
em conta os atritos entre a prancha e a água, bomba fragmenta-se em dois pedaços, A e B, de
AFA – EFOMM – EEAR| FÍSICA

ventos ou correntezas, e considerando para a massas respectivamente iguais a 2,0kg e 1,0kg.


prancha comprimento de 1,5m, calcule: Sabendo que, imediatamente após a explosão, o
a) a relação entre os módulos das quantidades de pedaço A se move para baixo, com velocidade de
movimento do homem e da prancha, enquanto intensidade 32m/s, determine:
o homem não alcança o ponto B.
b) a distância x do homem à escada, depois de a) a intensidade e o sentido da velocidade do
ter atingido o ponto B. pedaço B imediatamente depois da explosão;
c) o módulo da velocidade escalar média do b) o aumento da energia mecânica do sistema
homem em relação à escada e em relação à devido à explosão.
prancha, se, ao se deslocar de A até B, ele
gasta 2,0s. 26. Na figura, o bloco A (massa 4M) e a esfera B
(massa M) encontram-se inicialmente em repouso,
23. Novos sistemas de propulsão de foguetes e de com A apoiado em um plano horizontal:
sondas espaciais estão sempre sendo estudados
pela Nasa. Um dos projetos utiliza o princípio
de atirar e receber bolas de metal para ganhar
impulso. O sistema funcionaria da seguinte forma:
em uma estação espacial, um disco, girando,
atiraria bolas metálicas, a uma velocidade de
7200km/h. Uma sonda espacial as receberia e as
Largando-se a esfera B na posição indicada,
mandaria de volta ao disco da estação. Segundo
ela desce, descrevendo uma trajetória circular
pesquisadores, esse sistema de receber e atirar
(1/4 de circunferência) de 1,0m de raio e centro
bolas de metal poderia ser usado para dar o
em C. Desprezando todos os atritos, bem como
impulso inicial a naves ou sondas espaciais que
a influência do ar, e adotando IgI = 10m/s 2,
já estivessem em órbita.
determine os módulos das velocidades de A e de
(Adaptado de: Jornal Folha de S. Paulo.) B no instante em que a esfera perde o contato
com o bloco.

27. Uma caixa de massa 1,0 . 102kg, inicialmente


vazia, desloca-se horizontalmente sobre rodas
num plano liso, com velocidade constante de
4,0m/s. Em dado instante, começa a chover
Considere uma sonda espacial com massa de 1 e as gotas, que caem verticalmente, vão-se
tonelada, em repouso em relação a uma estação depositando na caixa, que é aberta.
espacial, conforme ilustra a figura acima. Suponha a) Qual a velocidade da caixa depois de ter
que a sonda receba, pela entrada E, uma bola de alojado 3,0 . 102kg de água?
10kg, atirada a 7 200km/h pelo disco da estação, e b) Se no instante em que a caixa contém
a devolva, pela saída 5, com um quinto do módulo 3,0 .102kg parar de chover e for aberto um
da velocidade inicial. Calcule, em m/s, o módulo orifício no seu fundo, por onde a água possa
da velocidade da sonda em relação à estação no escoar, qual será a velocidade final da caixa
instante em que a bola é devolvida. depois do escoamento de toda a água?
OS.:0424/13-Juliana
52
Prof. Rodrigo Lins

28. Na situação do esquema abaixo, um míssil move- 31. Em um cruzamento da cidade de Campina
se no sentido do eixo Ox com velocidade v0, de Grande, durante uma manhã de muita chuva,
módulo 40m/s. Em dado instante, ele explode, um automóvel compacto com massa de 1600kg
fragmentando-se em três partes A, B e C de que se deslocava de Oeste para Leste, com uma
massas M, 2M e 2M, respectivamente: velocidade de módulo 30m/s, colidiu com uma
picape (camionete) com massa de 2400kg que
se deslocava do Sul para o Norte, avançando o
sinal vermelho, com uma velocidade de módulo
15m/s, conforme a figura abaixo. Felizmente,
todas as pessoas, nesses veículos, usavam
cintos de segurança e ninguém se feriu. Porém
os dois veículos se engavetaram e passaram
Sabendo que, imediatamente após a explosão,
a se mover, após a colisão, como um único
as velocidades das partes B e C valem

AFA – EFOMM – EEAR| FÍSICA


corpo, numa direção entre Leste e Norte.
vB = vC = 110m/s2, determine as características da
Desprezando-se o atrito entre os veículos e a pista,
velocidade vetorial da parte A, levando em conta o
referencial Oxy. o módulo da velocidade dos carros unidos após a
colisão, em m/s, foi de:

29. O rojão representado na figura tem, inicialmente,


ao cair, velocidade vertical de módulo 20m/s. Ao
explodir, divide-se em dois fragmentos de massas
iguais, cujas velocidades têm módulos iguais e
direções que formam entre si um ângulo de 120°.

Dados: sen 30° = cos 60° = 0,50; cos 30° = sen


60°= 0,87.
a) 15
b) 16
c) 18
d) 20
e) 22

32. Duas bolas de gude idênticas, de massa m,


movimentam-se em sentidos opostos (veja a
O módulo da velocidade, em m/s, de cada figura) com velocidades de módulo v:
fragmento, imediatamente após a explosão, será:
a) 10
b) 20
Indique a opção que pode representar as
c) 30
velocidades das bolas imediatamente depois da
d) 40
colisão:
e) 50

30. Um navio que se encontra inicialmente em


repouso explode em três pedaços. Dois dos
pedaços, de massas iguais, partem em direções
perpendiculares entre si, com velocidades de
módulo 100km/h. Supondo que a massa do
terceiro pedaço seja o triplo da massa de um dos
outros dois, qual o valor aproximado do módulo de
sua velocidade imediatamente após a explosão?
53 OS.:0424/13-Juliana
Prof. Rodrigo Lins

33. Nas situações representadas nas figuras abaixo, 37. Dois patinadores de massas iguais deslocam-se
as partículas realizam colisões unidimensionais. numa mesma trajetória retilínea, com velocidades
Os módulos de suas velocidades escalares estão escalares respectivamente iguais a 1,5m/s e
indicados. Determine, em cada caso, o coeficiente 3,5m/s. O patinador mais rápido persegue o outro.
de restituição da colisão, dizendo, ainda, se a Ao alcançá-lo, salta verticalmente e agarra-se às
interação ocorrida foi elástica, totalmente inelástica
suas costas, passando os dois a deslocarem-se
ou parcialmente elástica.
com velocidade escalar v. Desprezando o atrito,
calcule o valor de v.

38. A figura abaixo apresenta os gráficos da velocidade


versus tempo para a colisão unidimensional
ocorrida entre dois carrinhos A e B:
AFA – EFOMM – EEAR| FÍSICA

34. No esquema abaixo, estão representadas as


situações imediatamente anterior e imediatamente
posterior à colisão unidimensional ocorrida entre Supondo que não existam forças externas
duas partículas A e B: resultantes e que a massa do carrinho A valha
0,2kg, calcule:
a) o coeficiente de restituição da colisão;
b) a massa do carrinho B.

Sendo conhecidos os módulos das velocidades


escalares das partículas, calcule a relação 39. Sobre uma mesa horizontal de atrito desprezível,
mA/mB entre suas massas. dois blocos A e B de massas m e 2m,
respectivamente, movendo-se ao longo de uma
reta, colidem um com o outro. Após a colisão, os
35. Os carrinhos representados nas figuras abaixo, ao
blocos se mantêm unidos e deslocam-se para a
percorrer trilhos retilíneos, colidem frontalmente.
Os módulos de suas velocidades escalares antes direita com velocidade V , como indicado na figura.
e depois das interações estão indicados nos O único esquema que não pode representar os
esquemas. Calcule, para as situações dos itens a e movimentos dos dois blocos antes da colisão é:
b, a relação m1/m2 entre as massas dos carrinhos
(1) e (2).

40. Duas bolas de boliche A e B, de massas iguais,


36. Uma locomotiva de massa 200t movendo-se
percorrem uma mesma canaleta retilínea onde
sobre trilhos retos e horizontais com velocidade
de intensidade 18,0km/h colide com um vagão de realizam um choque perfeitamente elástico. Se
massa 50t inicialmente em repouso. Se o vagão as velocidades escalares de A e B imediatamente
fica acoplado à locomotiva, determine a intensidade antes da colisão valem vA = 2,0m/s e vB = -1,0m/s,
da velocidade do conjunto imediatamente após a quais as velocidades escalares VA e vB de A e B
colisão. imediatamente depois da colisão?
OS.:0424/13-Juliana
54
Prof. Rodrigo Lins

41. Considere a montagem experimental representada


abaixo, em que a esfera 1 tem massa 2M e as
demais (2, 3, 4 e 5) têm massa M:

Admitindo-se que as colisões entre A, B e C


sejam unidimensionais e perfeitamente elásticas,
determine as velocidades escalares desses blocos
depois de ocorridas todas as colisões possíveis
Abandonando-se a esfera 1 na posição indicada, entre eles.
ela desce, chegando ao ponto mais baixo de
sua trajetória com velocidade v0. Supondo que 44. A figura representa a situação imediatamente
todas as possíveis colisões sejam perfeitamente anterior à colisão unidimensional entre duas
elásticas, podemos afirmar que, após a interação: partículas A e B:
a) a esfera 5 sai com velocidade 2v0.
b) as esferas 2, 3, 4 e 5 saem com velocidade

AFA – EFOMM – EEAR| FÍSICA


v0/ 2.
c) as esferas 4 e 5 saem com velocidade v0.
d) as esferas 2, 3, 4 e 5 saem com velocidade Sabendo que a massa de B é o dobro da de A
v0. e que o coeficiente de restituição da colisão vale
e) todas as esferas permanecem em repouso. 0,8, calcule as velocidades escalares de A e B
imediatamente após o choque.

42. Na figura, representamos uma mesa perfeitamente


45. A figura abaixo representa dois carrinhos A e B de
lisa e duas esferas A e B que vão realizar uma
massas m e 3m, respectivamente, que percorrem
colisão unidimensional e perfeitamente elástica.
um mesmo trilho retilíneo com velocidades
A esfera A tem massa m e, antes da colisão, se
escalares vA = 15m/s e vB = 5,0m/s:
desloca com velocidade constante de 60m/s. A
esfera B tem massa 2m e, antes da colisão, está
em repouso.

Se o choque mecânico que ocorre entre eles tem


coeficiente de restituição 0,2, quais as velocidades
escalares após a interação? Despreze os atritos.

Não considere a rotação das esferas. Sejam EA 46. Duas partículas 1 e 2, de massas respectivamente
a energia cinética de A antes da colisão e EB a iguais a 3,0kg e 2,0kg, percorrem uma mesma reta
energia cinética de B após a colisão. Indique a orientada com velocidades escalares v1 = 2,0m/s
opção correta: e v2 = –8;0m/s. Supondo que essas partículas
4 colidam e que o coeficiente de restituição do
a) EB = E A impacto seja 0,5, determine:
9
8 a) as velocidades escalares de 1 e de 2
b) EB = E A imediatamente após o impacto;
9
c) EB = E A b) a relação entre as energias cinéticas do
sistema (partículas 1 e 2) imediatamente após
9
d) EB = E A e imediatamente antes do impacto.
8
e) EB = 2E A
47. Uma esfera A, de massa 200g, colidiu frontalmente
com uma outra, B, de massa 300g, inicialmente em
repouso. Sabendo que A atingiu B com velocidade
43. Três blocos, A, B e C, de dimensões idênticas
escalar de 5,0cm/s e que esta última adquiriu,
e massas respectivamente iguais a 2M, M e M,
imediatamente após a colisão, velocidade escalar
estão inicialmente em repouso sobre uma mesa
de 3,0cm/s, determine:
horizontal sem atrito, alinhados num ambiente em
que a influência do ar é desprezível. O bloco A é a) o coeficiente de restituição para a colisão
então lançado contra o bloco B com velocidade ocorrida;
escalar de 9,0m/s, conforme indica a figura. b) o percentual de energia cinética dissipada por
efeito do impacto.
55 OS.:0424/13-Juliana
Prof. Rodrigo Lins

48. Um objeto de massa m1= 4,0kg e velocidade 51. A figura 1 abaixo ilustra um projétil de massa
escalar v1 = 3,0m/s choca-se com um objeto em m 1 = 20g disparado horizontalmente com
repouso, de massa m2 = 2,0kg. A colisão ocorre de velocidade de módulo v1 = 200m/s contra um bloco
modo que a perda de energia cinética é máxima, de massa m2 = 1,98kg, em repouso, suspenso na
mas consistente com o Princípio de Conservação vertical por um fio de massa desprezível. Após
da Quantidade de Movimento. sofrerem uma colisão perfeitamente inelástica, o
projétil fica incrustado no bloco e o sistema projétil-
a) Quais as velocidades escalares dos objetos bloco atinge uma altura máxima h, conforme
imediatamente após a colisão? representado na figura 2.
b) Qual a variação da energia cinética do
sistema?

49. Realiza-se, no laboratório, um experimento


em que são utilizados dois pêndulos iguais,
Desprezando-se a força de resistência do ar e
AFA – EFOMM – EEAR| FÍSICA

A e B, em cujos fios estão presas esferas de


adotando-se g = 10m/s2, resolva os itens abaixo.
massa de vidraceiro. Inicialmente, os pêndulos
encontram-se em repouso, dispostos conforme a) Calcule o módulo da velocidade que o sistema
ilustra o esquema: projétil-bloco adquire imediatamente após a
colisão.
b) Aplicando-se o Princípio da Conservação da
Energia Mecânica, calcule o valor da altura
máxima h atingida pelo sistema projétil-bloco
após a colisão.

52. Uma caixa de dimensões desprezíveis tem massa


Largando-se o pêndulo A, ele desce sem sofrer m = 2,0kg e encontra-se inicialmente em repouso
os efeitos do ar, indo colidir de modo totalmente sobre uma mesa horizontal, sem atrito. A partir do
inelástico com o pêndulo B. Podemos afirmar que instante t0 = 0, passa a agir sobre ela uma força
o percentual de energia mecânica dissipado nesse paralela à mesa, cuja intensidade varia em função
experimento, por efeito da colisão, vale: do tempo, conforme o gráfico abaixo:

a) 10%.
b) 25%.
c) 50%.
d) 75%.
e) 100%.
Admitindo que a força tenha direção constante e
que atue na caixa somente até o instante t = 6,0s,
50. O dispositivo representado na figura abaixo
determine:
denomina-se pêndulo balístico e pode ser
utilizado para a determinação da intensidade da a) o instante em que a caixa atinge velocidade
máxima;
velocidade de projéteis:
b) o módulo da velocidade da caixa nos instantes
t = 2,0s e t = 8,0s.

53. Numa importante final futebolística, um jogador


cobra um pênalti e a bola, depois de chocar-se
contra o travessão, sai numa direção perpendicular
Considere desprezíveis os pesos das hastes e o à do movimento inicial.
efeito do ar. Um projétil de massa m é disparado
horizontalmente com velocidade v0 contra o bloco
de massa M, inicialmente em repouso. O projétil
fica incrustado no bloco e o conjunto eleva-se
a uma altura máxima h. Sendo g o módulo da
aceleração da gravidade, determine, em função
de M, m, g e h, a intensidade de v0.
OS.:0424/13-Juliana
56
Prof. Rodrigo Lins

A bola, que tem 0,50kg de massa, incide no 57. Na figura, temos um plano horizontal sem atrito
travessão com velocidade de módulo 80m/s e e um bloco B, em repouso, com o formato de um
recebe deste uma força de intensidade média prisma. Uma pequena esfera A é abandonada
5,0 . 103N. Sabendo que o impacto da bola no do repouso, da posição indicada na figura, e,
travessão dura 1,0 . 102s, calcule: após uma queda livre, colide elasticamente
a) o módulo da velocidade da bola imediatamente com o prisma. Despreze o efeito do ar e adote
após o impacto; g = 10m . s–2.
b) a energia mecânica dissipada no ato do
impacto.

54. Um barco de massa M = 160kg encontra-se em


repouso na superfície das águas de um lago, no
Sabe-se que, imediatamente após a colisão,
qual não há correntezas. Dentro do barco está um
a esfera A tem velocidade horizontal. A massa

AFA – EFOMM – EEAR| FÍSICA


homem de massa m = 80kg, que em dado instante
do prisma B é o dobro da massa da esfera A.
salta, deixando o barco com velocidade de módulo
A velocidade adquirida pelo prisma B, após a
2,0m/s, paralela às águas e medida em relação
colisão, tem módulo igual a:
às margens do lago. Desprezando os atritos e o
efeito do ar, determine: a) 2,0m/s.
b) 4,0m/s.
a) o módulo da velocidade do barco após o salto
do homem; c) 8,0m/s.
b) o trabalho da força que o homem exerce no d) 16m/s.
barco, por ocasião do seu salto. e) 1,0m/s.

55. Um avião voando horizontalmente atira um projétil 58. Uma partícula de massa 2,0kg, que se desloca
de massa 8,0kg, que sai com velocidade de para a direita com velocidade de 9,0m/s, colide
5,0 . 102m/s relativa ao solo. O projétil é disparado de modo totalmente inelástico com outra partícula
na mesma direção e no mesmo sentido em que de massa 4,0kg, que se desloca para a esquerda
voa o avião. Sabendo que a massa do avião sem com velocidade de 6,0m/s. O módulo do impulso,
o projétil vale 12 toneladas, calcule, em km/h, em unidades do SI, aplicado à partícula de 2,0kg
o decréscimo na velocidade da aeronave em durante a colisão é:
consequência do tiro. a) 12.
b) 16.
c) 18.
56. Um artefato explosivo, inicialmente em repouso,
é detonado, fragmentando-se em quatro partes, d) 20.
A, B, C e D, de massas respectivamente iguais a e) 24.
3,0kg, 2,5kg, 2,0kg e 4,0kg. Despreze a perda de
massa do sistema no ato da explosão e admita 59. Uma bola de aço de massa igual a 300g
que os quatro fragmentos sejam lançados com desloca-se com velocidade de intensidade
velocidades contidas em um mesmo plano. No 90m/s para a direita sobre um plano horizontal
esquema abaixo, são fornecidas as características perfeitamente liso. Ela colide frontal e elasticamente
das velocidades vetoriais adquiridas por A, B e C. com uma outra bola idêntica, inicialmente em
repouso.

Qual o valor do impulso, em kg . m/s, exercido


Aponte a alternativa que melhor traduz as pela parede à direita sobre a segunda bola,
características da velocidade vetorial adquirida sabendo-se que a colisão entre ambas é
por D: perfeitamente elástica?
57 OS.:0424/13-Juliana
Prof. Rodrigo Lins

60. No brinquedo ilustrado na figura, o bloco de massa 62. Sobre uma mesa fixa, de altura 0,80m, está
m encontra-se em repouso sobre uma superfície conectada uma rampa perfeitamente polida em
horizontal e deve ser impulsionado para tentar forma de quadrante de circunferência de raio
atingir a caçapa, situada a uma distância x = 1,5m 45cm, conforme representa a figura. Do ponto A da
do bloco. Para impulsioná-lo, utiliza-se um pêndulo rampa, abandona-se uma partícula de massa m,
de mesma massa m. O pêndulo é abandonado de que vai chocar-se de modo perfeitamente elástico
uma altura h = 20cm em relação à sua posição com outra partícula de massa 2m, em repouso no
de equilíbrio e colide elasticamente com o bloco
ponto B, o mais baixo da rampa.
no instante em que passa pela posição vertical.
Considerandose a aceleração da gravidade com
módulo g = 10m/s2, calcule:
AFA – EFOMM – EEAR| FÍSICA

Sabendo que no local a influência do ar é


a) a intensidade da velocidade da esfera do
desprezível e que g = 10 m/s2, determine:
pêndulo imediatamente antes da colisão;
b) a intensidade da velocidade do bloco a) a intensidade da velocidade da partícula de
imediatamente após a colisão; massa 2m ao atingir o solo;
c) a distância percorrida pelo bloco sobre b) a altura, acima do tampo da mesa, atingida
a superfície horizontal, supondo que o pela partícula de massa m após a colisão com
coeficiente de atrito cinético entre o bloco e a partícula de massa 2m;
essa superfície seja µ = 0,20. Verifique se o c) a distância entre os pontos de impacto das
bloco atinge a caçapa. partículas com o solo.

61. J o ã o , n u m a t o d e g e n t i l e z a , e m p u r r a
63. Num circo, um homem-bala de massa 60kg é
horizontalmente uma poltrona (massa igual a
disparado por um canhão com velocidade V 0
10kg) para Maria (massa igual a 50kg), que a
de módulo 25m/s, sob um ângulo de 37° com
espera em repouso num segundo plano horizontal
a horizontal: Sua parceira, cuja massa é 40kg,
0,80m abaixo do plano em que se desloca João,
está em repouso numa plataforma localizada no
conforme indica a figura.
topo da trajetória. Ao passar pela plataforma, o
homem-bala e a parceira se agarram e vão cair
em uma rede de segurança, na mesma altura que
o canhão. Veja a figura fora de escala abaixo.

A poltrona é empurrada a partir do repouso


de A até B, ao longo de 4,0m, por uma força
constante F de intensidade 25N. Em B, ela é solta,
descendo uma pequena rampa e atingindo Maria
com velocidade de intensidade V, que se senta
rapidamente. Com isso, o sistema poltrona-Maria
Desprezando-se a resistência do ar e
passa a se deslocar com velocidade de intensidade
considerando-se sen 37° = 0,60, cos 37° = 0,80
V'. Desprezando-se os efeitos do ar e também
e g = 10m/s2, pode-se afirmar que o alcance A
os atritos sobre a poltrona e considerando-se
g = 10 m/s2, determine: atingido pelo homem é:

a) o trabalho da força aplicada por João sobre a a) 60m.


poltrona no percurso de A até B. b) 48m.
b) o valor de V. c) 36m.
c) o valor de V'. d) 24m.
OS.:0424/13-Juliana
58
Prof. Rodrigo Lins

64. Em uma autoestrada, por causa da quebra de uma 67. Uma porta que tem a sua metade inferior feita de
ponta de eixo, a roda de um caminhão desprende- madeira e sua metade superior feita de vidro tem
se e vai em direção à outra pista, atingindo um carro espessura constante e as dimensões indicadas
que vem em sentido oposto. A roda é lançada com na figura.
uma velocidade de 72km/h, formando um ângulo de
30° com a pista, como indicado na figura abaixo. A
velocidade do carro antes da colisão é de 90km/h; a
massa do carro é igual a 900kg e a massa da roda
do caminhão é igual a 100kg. A roda fica presa ao
carro após a colisão.
Sabendo que a massa da parte de vidro é 23
da massa da parte de madeira, determine as
coordenadas x e y do centro de massa da porta,
dadas pelo referencial Oxy.

AFA – EFOMM – EEAR| FÍSICA


68. Uma barra metálica é constituída pela junção
de dois cilindros A e B, coaxiais e de materiais
diferentes:

a) Imediatamente após a colisão, qual é a


componente da velocidade do carro na direção
transversal à pista?
b) Qual é a energia cinética do conjunto
carro-roda imediatamente após a colisão? Supondo que os dois cilindros tenham seções
Se for necessário, use: sen 30° = 0,50; transversais constantes e iguais e admitindo
cos 30° = 0,87. uniforme a distribuição de massas em cada um
deles, determine a posição do centro de massa
da barra.
65. Na figura a seguir, temos uma massa M = 132
gramas, inicialmente em repouso, presa a uma
69. A forma de uma raquete de tênis pode ser
mola de constante elástica K = 1,6 . 104N/m,
esquematizada por um aro circular homogêneo
podendo deslocar-se sem atrito sobre a mesa em
de raio R e massa m1, preso a um cabo cilíndrico
que se encontra. Atira-se um projétil de massa
homogêneo de comprimento L e massa m 2 .
m = 12 gramas, que encontra o bloco
Quando R = L/4 e m1= m2, a distância do centro
horizontalmente, com velocidade v0 = 200m/s,
de massa da raquete ao centro do aro circular vale:
incrustando-se nele.
R
a)
2
b) R
3R
c)
Qual é a máxima deformação que a mola 2
experimenta? d) 2R

66. Quatro partículas, P1, P2, P3 e P4, de massas 70. Um artista plástico elaborou uma escultura que
respectivamente iguais a 1,0kg, 2,0kg, 3,0kg e consiste em um disco metálico homogêneo de
4,0kg, encontram-se sobre um mesmo plano, espessura constante e raio R dotado de um furo
posicionadas em relação a um referencial Oxy, circular de raio = R/2, conforme representa a figura.
conforme a figura abaixo:

Levando-se em conta o referencial Oxy indicado,


Determine as coordenadas x e y do centro de determine as coordenadas do centro de massa da
massa do sistema. peça.
59 OS.:0424/13-Juliana
Prof. Rodrigo Lins

71. O esquema abaixo representa dois carrinhos A Gabarito dos Exercícios Propostos
e B, que percorrem uma reta orientada com as
velocidades indicadas:
01. 15m/s
02. 40N

03. | Fm | = mV
Sabendo que as massas de A e de B valem, ∆t
respectivamente, 4,0kg e 6,0kg, calcule a 04. 20000N.s
velocidade do centro de massa do sistema.
05. a) 20s b) 1250N
06. a) 18kg.m/s b) 360J
72. Uma bola de bilhar de 200g de massa é lançada 07. 11
com velocidade de módulo 1,0m/s contra outra
08. d
AFA – EFOMM – EEAR| FÍSICA

igual, inicialmente em repouso. Qual o módulo


da velocidade do centro de massa do sistema 09. 5m/s
constituído pelas duas bolas? 10. 0,4m/s
11. 10kg
12. 2min30s
73. Dois navios, N1 e N2, de massas respectivamente
iguais a 250t e 150t, partem de um mesmo ponto 13. c
e adquirem movimentos retilíneos perpendiculares
14. e
entre si. Sabendo que as velocidades de N1 e N2
têm módulos v1 = 32 nós e v2 = 40 nós, podemos m
15.
= vc v 0 cos θ
afirmar que o centro de massa do sistema terá M
velocidade de módulo: 16. 1m/s e 10m/s
a) 35 nós. 17. a) 0,5m/s b) –112,5J
b) 25 nós.
18. µ ≅ 0,205
c) 20 nós.
d) 5 nós. 19. a) 6m/s b) 54J
e) zero. t2 v2 3 2
20. a)= 1;= ; b)
t1 v1 2 3
74. Um conjunto de três partículas, todas de igual 21. a) 0,3m/s b) 1,5m
massa m, está situado na origem de um sistema 22. a) 1 b) 50cm c) 0,5m/s e 0,75m/s
de coordenadas xy. Em dado instante, uma delas
23. 24m/s
é atirada na direção x, com velocidade constante
vx = 9,0m/s, e outra é atirada, simultaneamente, 24. –1000m/s
na direção y, com velocidade constante
25. a) 4,0m/s para cima b) 432J
vy = 12m/s, ficando a terceira em repouso na
origem. Determine o módulo da velocidade do 26. 1m/s e 4m/s
centro de massa do conjunto. 27. a) 1m/s b) 1m/s
28. Imediatamente após a explosão, o fragmento A tem
velocidade na direção do eixo Ox, sentido oposto
ao do referido eixo e módulo de 20m/s.
29. d
30. aproximadamente 47km/h
31. a
32. e
33. a) e = 0,25; parcialmente elástica
b) e = 0; totalmente inelástica

OS.:0424/13-Juliana
60
Prof. Rodrigo Lins

c) e = 1; elástica 67. x = 0,4 e y = 0,9


d) e = 0,6; parcialmente elástica 68. 1/2L
e) e = 0; totalmente inelástica 69. c
34. 5/4 5R
=
70. x R=
e y
m m 6
=
35. a) 1 1= b) 1 0,6 71. 14m/s
m2 m2
36. 14,4km/h 72. 0,5m/s
37. 2,5m/s 73. b
38. a) 0,6 b) 0,2kg 74. 5m/s
39. d
40. vA' = –1,0m/s vB' = 2,0m/s

AFA – EFOMM – EEAR| FÍSICA


41. c
Gabarito dos Exercícios de Fixação
42. b
43. Bloco A: 1,0m/s; Bloco B: 4,0m/s; Bloco C: 12m/s 01. e
44. vA' = –3,0m/s vB' = 1,0m/s 02. a
03. c
45. a) 6,0m/s b) 8,0m/s
04. d
5
46. a) (1): –4,0m/s; (2): 1,0m/s b) 05. e
14
47. a) 0,5 b) 45% 06. d
48. a) 2,0m/s b) –6,0J 07. d
49. c 08. d
M+m 09. e
50. v 0 = 2gh
m
10. e
51. a) 2m/s b) 20cm
11. 1600kg.m/s
52. a) t = 6,0s b) 2,0m/s e 6,0m/s
12. a) 200N.s b) 30m/s
53. a) 60m/s b) 700J
13. a) 60N b) 2cm
54. a) 1m/s b) 80J
14. 14s
55. 1,2km/h
15. 20m/s
56. a
57. a
58. d
59. 54kg.m/s
60. a) 2,0m/s b) 2,0m/s
c) 1,0m, e o bloco não atinge a caçapa.
61. a) 1,0 . 102J b) 6,0m/s c) 1,0m/s
62. a) 4,5m/s b) 5,0cm c) 40cm
63. b
64. a) 1,0m/s b) ≃2,16 . 105J
65. 5cm
66. x = 2 e y = 1,1

61 OS.:0424/13-Juliana

Você também pode gostar